• Shuffle
    Toggle On
    Toggle Off
  • Alphabetize
    Toggle On
    Toggle Off
  • Front First
    Toggle On
    Toggle Off
  • Both Sides
    Toggle On
    Toggle Off
  • Read
    Toggle On
    Toggle Off
Reading...
Front

Card Range To Study

through

image

Play button

image

Play button

image

Progress

1/166

Click to flip

Use LEFT and RIGHT arrow keys to navigate between flashcards;

Use UP and DOWN arrow keys to flip the card;

H to show hint;

A reads text to speech;

166 Cards in this Set

  • Front
  • Back

129. After completion of chemoradiation for anal canal cancer:
(A) an assessment at 4 weeks by MRI is performed.
(B) biopsies are performed even in clinical complete responses to rule out occult disease.
(C) persistent disease at 4 weeks is followed to document regression.
(D) for progressive local disease, chemotherapy is preferred over salvage surgery.


Key: C
Rationale: Typically post-treatment monitoring would begin at 8-12 weeks of therapy by clinical exam (DRE, anoscopy and clinical nodal evaluation). Imaging at 4 weeks would likely be misleading and wouldn’t be considered. A clinical complete response wouldn’t be biopsied – it would continue to be followed (NCCN guidelines). The regression can indeed take quite some time with some studies showing continued regression >20 weeks after completion of chemoradiation. Most would agree that in the setting of histological documented local progression that a local therapy (such as APR) would be preferred over systemic therapy.
References: NCCN Guidelines.


245. A 3 cm squamous cell cancer of the anal canal with a 2 cm left inguinal node is staged as:
(A) T2N2 – stage IIB.
(B) T2N2 – stage IIIB.
(C) T3N1 – stage IIB.
(D) T3N1 – stage IIIB.


Key: B
Rationale: A 3 cm tumor would be T2 (2-5cm) and a unilateral inguinal node would be N2. The correct stage for a T2N2 is IIIB.
References: AJCC 7th Edition (2009).


148. According to the SHARP trial, what is the response rate to sorafanib among patients treated with hepatocellular carcinoma?
(A) 2%
(B) 5%
(C) 12%
(D) 20%


Key: A
Rationale: In this multicenter, phase 3, double-blind, placebo-controlled trial, 602 patients with advanced hepatocellular carcinoma who had not received previous systemic treatment were randomized to receive either sorafenib (at a dose of 400 mg twice daily) or placebo. Primary outcomes were overall survival and the time to symptomatic progression. Median overall survival was 10.7 months in the sorafenib group and 7.9 months in the placebo group (hazard ratio in the sorafenib group, 0.69; 95% confidence interval, 0.5.
References: Llovet et al. “Sorafenib in Advanced Hepatocellular Carcinoma.” NEJM 359(4):378-90, 2008.


164. Which of the following lymph nodes is considered an N1 lymph node in gastric cancer?
(A) Subpyloric
(B) Common hepatic
(C) Celiac
(D) Splenic

Key: A
Rationale: The first eschelon of lymph nodes (N1) are the perigastric lymph nodes including the R and L paracardial LN, greater and lesser curvature LN, and the suprapyloric and subpyloric LNs. The second eschelon of lymph node drainage (N2) includes the celiac artery and its branches.
References: Hartgrink et al. JCO 22(11): 2069-2077, 2004.

196. Which of the following lymph nodes should be electively irradiated for a locally advanced middle to high rectal cancer?
(A) Internal iliac and peri-rectal
(B) Internal iliac, pre-sacral, and peri-rectal
(C) internal iliac, external iliac, pre-sacral, and peri-rectal
(D) Internal iliac, external iliac, inguinal, pre-sacral, and peri-rectal

Key: B
Rationale: For a middle-to-high rectal cancer, the most appropriate choice is B. Pre-sacral lymph nodes are a site at high risk of failure, whereas external iliac and inguinal lymph nodes are not likely to be involved with a cancer at this level.
References: Myerson R, Int J Radiat Oncol Biol Phys. 2009 July 1; 74(3) pp. 824-30.

155. What is the MOST appropriate management for a mid-rectal T3N1M0 cancer?
(A) Pelvic radiation alone
(B) Low anterior resection followed by chemoradiation
(C) Abdomino-perineal resection
(D) Chemoradiation followed by low anterior resection

Key: D
Rationale: For locally advanced rectal cancer, the most appropriate choice is chemoradiation followed by surgical resection, in this case low anterior resection with total mesorectal excision. The superiority of pre-operative vs. post-operative chemoradiation was demonstrated in the study by Sauer and colleagues with improved local control (6 vs. 13 percent) and decreased toxicity (27 vs. 40 percent grade 3-4) as compared to postoperative treatment. The treatment of locally advanced rectal cancer with surgery alone will result in higher rates of local recurrence. Radiation alone is not a standard definitive approach.
References: Sauer R, et al. New England Journal of Medicine. Oct 21, 2004; 351(17) pp: 1731-40. Kapiteijn E, et al. New England Journal of Medicine. Aug 30 2001; 345(9) pp: 638-46.

156. Which lymph nodes have the lowest likelihood of involvement for tumors in the distal stomach?
(A) Periduodenal
(B) Splenic hilar
(C) Peripancreatic
(D) Hepatoduodenal

Key: B
Rationale: Tumors in the distal stomach have the highest likelihood of travelling to the periduodenal, peripancreatic, and hepatoduodenal stations. They have the lowest likelihood of involving the periesophageal, mediastinal and splenic hilar lymph nodes.
References: Tepper et al. Seminars in Radiation Oncology 12(2): 187-195, 2002.



22. Which of the following is TRUE concerning the use of adjuvant radiation in colon cancer?
(A) It is routine, as colon cancer has significantly higher local recurrence rates than rectal cancer.
(B) The INT 0130 trial showed adjuvant chemoradiation significantly improved survival compared to chemotherapy alone.
(C) The INT 0130 trial showed adjuvant chemoradiation significantly improved disease-free survival compared to chemotherapy alone.
(D) There is no prospective evidence supporting the use of adjuvant radiation in colon cancer patients.

Key: D
Rationale: Adjuvant radiation is not typically used in resected colon cancer. Colon cancer has lower rates of local recurrence than rectal cancer (matched for stage) due to the location and anatomy of the rectum. The INT0130 randomized patients (any T4; or T3 Node positive patients of the ascending and descending colon) to 5FU+Levamisole with or without radiation. The trial was closed to low accrual and was underpowered but did not show any significant differences in OS or DFS. The only data that supports the use of radiation in colon cancer is from retrospective series.
References: INT 0130 (Martenson et al., JCO 22(16): 3277-3283).

257. Gastrointestinal Tumor Study Group (GITSG) trial GI9173 randomized patients
with resected pancreatic cancer to either adjuvant combined radiation and 5FU or no
further treatment following surgery. The median survival in patients receiving surgery
alone was closest to which one of the following?
A. 6 months
B. 12 months
C. 18 months
D. 24 months

Answer 257 is B. The median survival was 11 months for patients in the surgery-alone
arm, vs 20 months for patients who received radiation and 5FU.

258. In the randomized trial of surgery followed by 5FU and radiation vs surgery alone
for patients with resected gastric cancer reported by MacDonald et al, the addition of
adjuvant therapy resulted in an absolute improvement in 3-year overall survival of
roughly:
A. 10%
B. 20%
C. 30%
D. 40%

Answer 258 is A. Patient survival in the surgery-alone arm was 41%, vs 50% in the arm
receiving adjuvant chemo-radiation.

261. In general, which one of the following tumors has the lowest ratio of deaths per year
to new cases per year?
A. Stomach
B. Esophagus
C. Gallbladder
D. Anus

Answer 261 is D. According to the Jan/Feb 2004 issue of CA-A Cancer Journal, the
ratios of deaths to cases for choices A through D are .52, .93, .51 and .145.

264. Regarding treatment outcomes observed in esophageal clinical trials involving
primary chemoradiotherapy, which one of the following is INCORRECT?
A. RTOG 85-01 demonstrated that chemotherapy with 5FU and cisplatin combined with
radiation therapy produced statistically superior overall survival when compared with
radiation alone
B. RTOG 85-01 demonstrated 45% risk of locoregional disease persistence or
recurrence after definitive chemoradiotherapy
C. Most esophageal adenocarcinomas arise in the setting of Barrett esophagus
D. Endoscopic (including biopsy) reevaluation after a course of preoperative
chemoradiotherapy provides an accurate estimate of the pathologic complete response
rate

Answer 264 is D. Studies of endoscopic restaging versus surgical pathological staging
have demonstrated that endoscopy has a high false-negative rate for detecting persisting
tumor.

265. Regarding gastric cancer, which one of the following is INCORRECT?
A. The United States has one of the lowest rates of gastric cancer among developed
countries
B. The Intergroup Gastric adenocarcinoma Trial 0116 randomized patients after surgery
to either observation vs. preconcurrent and postradiation 5FU with leucovorin
C. Local, regional, and distant failure rates were all statistically improved in the
postsurgical chemoradiotherapy arm vs. the postsurgical observation arm of the
Intergroup Trial 0116
D. Li-Fraumeni syndrome is a risk factor for gastric cancer

Answer 265 is C. Local and regional failure rates were improved in the
chemoradiotherapy arm, but distant failure rates were not.

229. After esophagectomy is performed, the addition of radiation therapy has been shown
to achieve which of the following results for patients with esophageal cancer?
A. Improve local control
B. Improve overall patient survival
C. Decrease the incidence of distant metastases
D. Produce unacceptable toxicity

Answer 229 is A. Three trials of post-operative radiation showed no effect on overall survival.
Toxicity was acceptable. Local control was improved, particularly in node-negative patients, e.g.,
Teniere, Surg Gynecol Obstet. 1991, 173:123-30

237. Which of the following statements about pancreatic cancer is true?
A. K-ras gene mutations are associated with approximately 90% of patients with
pancreatic adenocarcinomas.
B. Approximately 50% of patients have resectable tumors at initial diagnosis.
C. Approximately 30% of patients who do not receive adjuvant therapy experience
locoregional relapse as a component of failure after undergoing a Whipple
procedure.
D. The sensitivity rate of CT scans to detect tumor resectability is approximately 30%.

Answer 237 is A. Modern, contrast enhanced CT scanning can correctly predicts resectability
about 70% of the time. Local/regional relapse as a component of failure after a Whipple
procedure in non-adjuvantly-treated patients is about 70%.

241. Which of the following statements about hepatocellular carcinoma is true?
A. Pattern of recurrence for hepatocellular carcinoma has a higher probability of a localregional
component than a metastatic component.
B. Analysis of α-fetoprotein serum levels is useful for determining prognosis.
C. Jaundice is an uncommon sign of the disease at presentation.
D. Orthotopic liver transplantation has not been shown to improve disease-free
survival or overall survival for patients.

Answer 241 is A. AFP levels of 20ng/ml in a patient with a liver mass are highly suggestive of
HCC, while levels greater than 500 are diagnostic. Levels greater than 2000 are associated with a
particularly poor prognosis. Jaundice is a common presenting sign. Orthotopic liver
transplantation has been shown to improve disease-free survival.

248. Which of the following statements about the German Rectal Cancer Study Group trial
that compared preoperative to postoperative chemoradiotherapy for rectal cancer is true?
A. A higher percentage of patients receiving preoperative versus postoperative
chemoradiotherapy were able to complete the prescribed course of therapy.
B. There were more grade-3 or grade-4 acute and chronic GI toxicities in the preoperative
versus the
postoperative adjuvant group.
C. Overall patient survival was improved in the preoperative versus the postoperative
group.
D. Local tumor control was the same in both groups.

Answer 248 is A. The Sauer study was a large, randomized trial using modern, full dose
radiotherapy and continuous infusion 5-FU that found favorable results for the preoperative arm
in terms of toxicities, treatment compliance and local control. It did not report better disease free
or overall survival. One speculation is that the observed better local control might relate to the
better tolerability and, therefore, therapy compliance in the preoperative arm.

250. Which of the following statements about anal cancer is true?
A. About 40,000 cases of anal cancer are diagnosed in the United States each year.
B. Malignant melanoma constitutes about 2% of all diagnosed anal cancers.
C. Liver and lung metastases are found in about 25% of patients at diagnosis.
D. Clinically detectable inguinal lymph node metastasis is found in 50% of patients with
squamous cell carcinoma at diagnosis.

Answer 250 is B. Five to 10% of patients have extra-pelvic metastatic disease at time of
diagnosis, most commonly in the liver and lung. About 4,000 cases of anal cancer are diagnosed
in the United States each year. Clinically detectable inguinal lymph node metastasis is found in
10% of squamous cell cases at the time of diagnosis

7. In RTOG 8501, what was the 5-year overall survival rate for patients who had esophageal cancer and received radiation therapy alone?
A. 00%
B. 10%
C. 25%
D. 35%

Correct answer is A. RATIONALE: This is a landmark study that established the superiority of combined chemoradiation therapy to radiation therapy alone for esophageal cancer. Reference: Herskovic. New England Journal of Medicine (NEJM). 1992;326:1593.

19. Which of the following diagnostic tests is most useful for determining the tumor stage of esophageal cancer?
A. Esophagogastroduodenoscopy
B. Endoscopic ultrasound
C. PET scan
D. CT scan

Correct answer is B. RATIONALE: Endoscopic ultrasound (EUS) is the most useful diagnostic test for determining the tumor (T) stage of esophageal cancer.

33. What is the recommended dose for definitive radiation treatment of esophageal cancer, according to the Intergroup 0123 study?
A. 45 Gy
B. 50.4 Gy
C. 54 Gy
D. 59.4 Gy

Correct answer is B. RATIONALE: In the Intergroup 0123 study, patients were randomized to 64 vs. 50.4 Gy. The lower dose arm had a higher median survival of 17.5 months vs. 12.9 months. Reference: Journal of Clinical Oncology. 2002;20:1167–74.

80. According to the CLASSIC trial, which of the following therapies improved disease-free survival among patients with gastric cancer?
(A) Perioperative chemotherapy with epirubicin, cisplatin, and 5-FU
(B) Postoperative chemoradiation with 5-FU
(C) Preoperative chemotherapy with epirubicin, cisplatin, and 5-FU and adjuvant chemoradiation
(D) Postoperative capecitabine and oxaliplatin

Key: D
Rationale: The capecitabine and oxaliplatin adjuvant study in stomach cancer (CLASSIC) study was a phase III randomized trial performed in Asia. Patients with stage II-IIIB gastric cancer who had had curative D2 gastrectomy were randomly assigned to receive adjuvant chemotherapy of eight 3-week cycles of oral capecitabine (1000 mg/m2 twice daily on days 1 to 14 of each cycle) plus intravenous oxaliplatin (130 mg/m2 on day 1 of each cycle) for 6 months or surgery only. The 3-year disease-free survival was significantly improved among patients treated with chemotherapy compared to surgery (74% vs. 59%, p<0.0001). Adjuvant capecitabine plus oxaliplatin treatment after curative D2 gastrectomy should be considered as a treatment option for patients with operable gastric cancer.
References: Bang et al.

81. What is the MOST appropriate next step for a patient with a localized adenocarcinoma at 8 cm from the anal verge?
(A) Chemoradiation
(B) Abdominoperineal resection
(C) Low anterior resection
(D) Endorectal ultrasound or pelvic MRI

Key: D
Rationale: As this patient has not yet been staged adequately, it is not yet clear what the proper management will be. Assessment of the T and N stage can be performed via either endorectal ultrasound or MRI of the pelvis. For patients with T3/4 or N+ disease, pre-operative chemoradiation is recommended.

46. Which of the following types of esophageal cancer has had the greatest increase in incidence over the past 40 years?
A. Squamous cell carcinoma in Caucasian men
B. Squamous cell carcinoma in African American men
C. Adenocarcinoma in Caucasian men
D. Adenocarcinoma in African American men

Correct answer is C. RATIONALE: Over the past 40 years, there has been a very dramatic increase in esophageal adenocarcinoma in Caucasian men, which has not yet been able to be explained. Reference: Devesa, et al. Cancer. 1998;83:2049-53.

56. According to Intergroup Trial 0116, adjuvant chemotherapy and radiation therapy after gastrectomy were more likely than gastrectomy alone to:
A. result in an unacceptable level of toxicity in the majority of patients.
B. result in a statistically significant decrease in the rate of distant metastases.
C. result in a statistically significant improvement in overall patient survival.
D. be most beneficial after D2 lymph node dissections.

Correct answer is C. RATIONALE: Overall survival increased from 41% to 50%, with a significant p value. Reference: New England Journal of Medicine (NEJM). 2002;345(10):725-30.

81. Which of the following factors has contributed most to the decline in incidence of distal gastric cancers in the United States?
A. Use of proton pump inhibitors
B. Improved food storage and preservation
C. Improved detection of Helicobacter pylori
D. Routine upper endoscopy screening of patients at 50 years of age

Correct answer is B. RATIONALE: Epidemiologic studies suggest that the routine use of refrigeration and the consequent avoidance of smoking and salting for food preservation is the major reason behind the decline in gastric cancer in the United States.

84. Which of the following chemotherapy regimens is most appropriate in the treatment of gastric cancer?
A. Cisplatin and VP-16
B. Gemcitabine and capecitabine
C. Epirubicin, cisplatin, and 5-FU
D. Cyclosphosphamide, methotrexate, and 5-FU

Correct answer is C. RATIONALE: Epirubicin, cisplatin, and 5-FU (ECF) comprise the chemotherapy regimen used in the MAGIC trial, which showed a benefit to perioperative chemotherapy, and in the ongoing Intergroup trial, which randomizes ECF to 5-FU/LV as given in Intergroup 0116.

96. What is the stage of a 5-cm gastric tumor that extends into, but not through, the muscularis propria with 16 of 23 positive lymph nodes?
A. I
B. II
C. III
D. IV

Correct answer is D. RATIONALE: In gastric cancer, involvement of >15 lymph nodes is classified as N3, which corresponds to stage IV.

110. What is the most common histology of tumors involving the terminal ileum?
A. Sarcoma
B. Carcinoid
C. Lymphoma
D. Adenocarcinoma

Correct answer is B. RATIONALE: Carcinoid is the most common histology of tumors involving the terminal ileum.

124. Which of the following types of cancer most commonly occurs in the GI tract in the U.S. population?
A. Colon
B. Rectal
C. Esophageal
D. Gastric

Correct answer is A. RATIONALE: There are approximately 150,000 cases of colon cancer per year, significantly more than the other choices. Reference: CA: A Cancer Journal for Clinicians. 2006;56:106-130.

137. Which of the following stages of colon cancer is most likely to benefit from adjuvant radiation therapy?
A. Stage T4N0 tumor of the sigmoid colon with invasion into the pelvic sidewall
B. Stage T3N2 tumor of the transverse colon
C. Stage T2N1 tumor of the descending colon
D. Stage T2N0 tumor of the appendix

Correct answer is A. RATIONALE: Although the only randomized trial of radiation therapy vs. no radiation therapy for colon cancer did not demonstrate an advantage to treatment, several retrospective studies and first principles of radiation suggest a T4 tumor with clear invasion into adjacent organs would be the most likely of the above choices to benefit from adjuvant therapy.

155. Approximately what percent of colorectal cancers are due to hereditary nonpolyposis colorectal cancer (HNPCC)?
A. 01%
B. 05%
C. 15%
D. 25%

Correct answer is B. RATIONALE: It is suggested that "up to 6%" of colorectal cancers are due to hereditary nonpolyposis colorectal cancer (HNPCC). Reference: Gastroenterology. 1993 (May);104(5):1535-49.

171. Which of the following types of colonic polyp is most likely to progress to carcinoma?
A. Hyperplastic polyp
B. Tubular adenoma
C. Tubulovillous adenoma
D. Villous adenoma

Correct answer is D. RATIONALE: Villous adenomas have a 15% to 25% chance of progressing to carcinoma.

187. Which of the following statements about patients who have HIV and anal cancer is FALSE?
A. HAART has been shown to reduce the severity and incidence of cancer-related toxicity.
B. The risk of increased toxicity with chemotherapy and radiation therapy is greater in patients with CD4 cell counts that are less than 200.
C. Patients with HIV are at increased risk for toxicity associated with chemotherapy and radiation therapy.
D. Patients with HIV are at increased risk for the development of anal intraepithelial neoplasia and squamous cell carcinoma of the anal canal.

Correct answer is A. RATIONALE: HAART has not been shown to reduce the severity and incidence of cancer-related toxicity.

4. According to the German Rectal Study, the 5-year risk of local recurrence for a patient with a T3-4, N+ rectal cancer receiving pre-operative chemoradiation followed by surgical resection is:
(A) 1%.
(B) 6%.
(C) 13%.
(D) 25%.

Key: B
Rationale: The local recurrence rate after pre-operative chemoradiation followed by surgical resection for a locally advanced rectal cancer is 6%.
References: Sauer R, et al. New England Journal of Medicine. Oct 21, 2004; 351(17) pp. 1731-40.

301. Based on the multi-institutional phase I/II trial of SBRT (Rusthoven et. al. (JCO 2009)) for liver metastases, what mL volume of normal liver should be spared?
(A) 100
(B) 250
(C) 700
(D) 1250

Key: C
Rationale: The protocol dose constraints for normal liver (total liver minus cumulative GTV) specified that a minimum volume of 700 mL should receive a total dose less than 15 Gy. From the surgical literature, we know that 75% to 80% of a non-cirrhotic liver can be safely resected. Assuming that the average liver is 2000 mL, then 25% would be 500 mL. Conservatively, they required that 700 mL should be spared (i.e. should receive less than 15 Gy).
References: Rusthoven et al. “Multi-institutional Phase I/II Trial of Stereotactic Body Radiation Therapy for Liver Metastases.” JCO 27(10): 1572-78, 2009.

293. Compared to radiation alone, chemoradiation in anal cancer improves:
(A) overall survival.
(B) colostomy-free survival.
(C) distant metastases-free survival.
(D) disease-free survival.

Key: B
Rationale: Both trials showed a statistically significant improvement in local control as well as in colostomy free survival. Overall survival was not statistically different between the two arms. Both trials used 5FU + Mitomycin C in the chemoradiation arms.
References: UKCCCR, Lancet 348:1049-54, 1996. EORTC, Bartelink et al., JCO 15(5):2040-49, 1997.

72. In the recently published CROSS study for esophageal cancer, patients were randomized to surgery alone versus neoadjuvant chemoradiation with:
(A) cisplatin and 5-FU.
(B) cisplatin and capecitabine.
(C) carboplatin and paclitaxel.
(D) carboplatin and cetuximab.

Key: C
Rationale: The CROSS study randomized 368 patients to receive either surgery alone or weekly carboplatin and paclitaxel for 5 weeks with concurrent radiotherapy. Preoperative chemoradiation was found to improve overall survival among potentially curable esophageal or GE junction cancers.
References: Van Hagen et al. NEJM 366(22): 2074-84, 2012.

201. Which of the following tumor stages is most likely to be associated with a patient who has a 3-cm squamous cell cancer of the anus with direct rectal wall invasion?
A. T1
B. T2
C. T3
D. T4

Correct answer is B. Reference. AJCC Cancer Staging Manual, 6th edition.

213. Which of the following statements about pancreatic cancer is FALSE?
A. Approximately 10% to 20% of pancreatic cancers are associated with hereditary factors.
B. New-onset diabetes mellitus may be the first clinical feature in 10% of patients.
C. Tumors of the pancreatic head arise to the right of the superior mesenteric vein-portal vein confluence and include tumors of uncinate origin.
D. The most common physical finding at initial presentation is Courvoisier's sign.

Correct answer is D. RATIONALE: The most common physical finding at initial presentation is jaundice.

231. In what region are cholangiocarcinomas most commonly located?
A. Distal
B. Proximal
C. Intrahepatic
D. Hilar

Correct answer is D. RATIONALE: Sixty-seven percent of cholangiocarcinomas occur in the hilar region.

247. Which of the following conditions is the most common risk factor for gallbladder cancer?
A. Cholelithiasis
B. Coledochal cysts
C. Primary sclerosing cholangitis
D. Liver fluke infestation

Correct answer is A. RATIONALE: Approximately 75% to 98% of patients with gallbladder cancer have cholelithiasis.

267. Which of the following statements about patients with primary sclerosing cholangitis (PSC) and cholangiocarcinoma is true?
A. Hepatobiliary cancer will develop in 25% to 30% of patients with PSC.
B. Cholangiocarcinoma tends to occur at an older age in patients who have PSC.
C. Approximately one third of cholangiocarcinomas occur within 10 years of diagnosis of PSC.
D. Patients who have both conditions often have a worse prognosis.

Correct answer is D. RATIONALE: Patients with primary sclerosing cholangitis and cholangiocarcinoma have a poor prognosis, because they often have multifocal cancer and underlying liver dysfunction.

305. Which of the following statements about the fibrolamellar variant of hepatocellular cancer is true?
A. It affects men and women equally.
B. It occurs more frequently in elderly persons.
C. It is commonly associated with cirrhosis of the liver.
D. Patients are less likely to present with positive lymph nodes.

THIS ITEM WAS PARTIALLY SCORED (BOTH OPTIONS A & D SCORED CORRECT). RATIONALE: No clear sex predominance is reported in fibrolamellar hepatocellular cancer.

320. Which of the following types of cancer is most frequently associated with isolated hepatic metastases at presentation?
A. Colorectal
B. Esophageal
C. Gastric
D. Lung

Correct answer is A. RATIONALE: Colorectal cancer is the most frequent type of malignancy that is associated with isolated hepatic metastasis at presentation.

322. What percent of patients with hepatocellular cancer are candidates for curative resection?
A. 75%
B. 50%
C. 20%
D. 05%

Correct answer is C. RATIONALE: In high-incidence regions of the world, only 10% to 15% of newly diagnosed patients are candidates for standard resection; whereas in low incidence areas, between 15% and 30% of patients are potential candidates for resection.


343. What percent of palpable inguinal nodes in patients with anal cancer are due to reactive hyperplasia?
A. 10%
B. 25%
C. 50%
D. 66%

Correct answer is C. RATIONALE: Fifty percent of palpable inguinal lymph nodes in patients with anal cancer are due to reactive hyperplasia.

7. For an adenocarcinoma at the gastroesophageal junction, involved celiac lymph nodes
would be classified as stage:
A. M1a.
B. M1b.
C. N1.
D. N2.

Correct answer is C. There is a discrepancy for item 7. The correct answer is A, not C, based upon the AJCC Cancer Staging Manual, 6th edition. This change does not affect the scores. RATIONALE: Celiac is M1a for distal intrathoracic esophagus.

19. Which of the following statements about adjuvant therapy for pancreatic cancer is true?
A. Gemcitabine improves disease-free survival.
B. Radiation therapy must be given to improve survival.
C. According to the ESPAC-1 trial, chemoradiation improved survival.
D. The Virginia Mason trial demonstrated lower than expected survival rates.

Correct answer is A. RATIONALE: The recent KONCO trial demonstrated a significant
improvement in disease-free survival for patients with pancreatic cancer who received adjuvant gemcitabine. The role of radiation therapy in this setting is still not clear, since none of the phase III studies today (except for the original GITSG trial) demonstrated a survival advantage. The Virginia Mason trial had a surprisingly high survival rate. The problem was excessive toxicity.

24. Postoperative chemoradiation administered to patients with gastric cancer improves:
A. distant metastases, locoregional control, and survival.
B. distant metastases and survival, but not locoregional control.
C. locoregional control and survival, but not distant metastases.
D. locoregional control, but not survival.

Correct answer is C. Reference: Macdonald. New England Journal of Medicine. Sept. 6, 2001;345(10):725-30.

42. Which of the following statements about treatment for a stage T2N0 squamous cell
carcinoma of the anal canal is true?
A. Cisplatinum with 5-FU is equally as effective as mitomycin C with 5-FU when
administered concurrently with radiation therapy.
B. The inguinal lymph nodes should be treated to 50.4 Gy with standard
chemotherapy.
C. Concurrent chemoradiation with mitomycin C and 5-FU are appropriate.
D. Radiation therapy of 50.4 Gy to 54 Gy alone delivered to the primary site and pelvic
lymph nodes is appropriate.

Correct answer is C. RATIONALE: The most recent phase III trial demonstrated inferiority of cisplatinum and 5-FU compared to mitomycin C and 5-FU when administered concurrently with radiation therapy. Squamous-cell carcinomas of the anal canal are exquisitely sensitive to chemoradiotherapy. Thus, a dose of 50.4 Gy is adequate for a 3-cm tumor when combined with mitomycin C and 5-FU, but not when radiation therapy is used alone. The appropriate prophylactic dose to the inguinal lymphatics for a stage T2N0 cancer is approximately 36 Gy.

63. What is the preoperative regimen used in the Swedish and Dutch rectal studies?
A. 5 Gy ラ5 fractions alone
B. 5 Gy ラ5 fractions with 5-FU
C. 3 Gy ラ10 fractions with capecitabine
D. 1.8 Gy ラ28 fractions with 5-FU

Correct answer is A. References: Kapiteijn E. New England Journal of Medicine, August 30, 2001;Vol. 345, No. 9. Folkesson J. Journal of Clinical Oncology. 2005;23:5644-5650.

70. Small bowel cancer is most likely to arise in which of the following locations?
A. Ileum
B. Jejunum
C. Ileocecal valve
D. Duodenum

Correct answer is D. RATIONALE: Fifty percent of small bowel cancers arise in the duodenum.

118. Occult bleeding is detected on what percent of screening fecal occult blood tests?
A. 10%
B. 25%
C. 50%
D. 80%

Correct answer: ABCD. RATIONALE: Occult bleeding is detected on screening fecal occult blood test (FOBT) in 26% of cases.

132. What percent of patients with colorectal cancer have other synchronous lesions, most of which are adenomatous polyps?
A. 75%
B. 50%
C. 30%
D. 10%

Correct answer is C. RATIONALE: Thirty percent of patients with colorectal cancer have synchronous lesions, usually adenomatous polyps.

145. Which of the following statements about assessment of the response to chemoradiation for squamous cell carcinoma of the anal canal is true?
A. The optimal time for biopsy is 4 to 6 weeks after treatment.
B. A biopsy should be performed after the end of treatment if the tumor has not
responded completely.
C. It is essential to document a complete pathological response immediately after
completion of chemoradiation.
D. After chemoradiation, the persistence of tumor cells within the primary site for up to
12 weeks does not indicate treatment failure.

Correct answer is D. RATIONALE: After chemoradiotherapy, tumor cells can persist within the primary site for up to 12 weeks, even in patients in whom the tumor is eventually eradicated. Early biopsy is contraindicated and could lead to morbidity. Biopsy should be obtained when there is clinical suspicion of progression.

151. Which of the following perioperative chemotherapy agents were used in the MAGIC
study for gastric cancer?
A. Etoposide, cisplatin, 5-FU
B. Epirubicin, cisplatin, and 5-FU
C. Cisplatin and 5-FU only
D. 5-FU alone

Correct answer is B. Reference: Cunningham D. New England Journal of Medicine.
2006;355:1.

103. In the recently published CROSS study for esophageal cancer, the pathologic complete response rate among patients treated with preoperative therapy is:
(A) 8%.
(B) 15%.
(C) 29%.
(D) 40%.

Key: C
Rationale: Among patients randomized to receive preoperative chemoradiation, a pathologic complete response was achieved in 29% (47 of 161) who underwent resection.
References: Van Hagen et al. NEJM 366(22): 2074-84, 2012.

158. Which of the following chemotherapeutic agents is most effective for treatment of
colorectal carcinoma?
A. Irinotecan
B. Bevacizumab
C. 5-Fluorouracil
D. Oxaliplatin

Correct answer is C. RATIONALE: The most useful chemotherapeutic agent for colorectal carcinoma is 5-fluorouracil (5-FU), which is an antimetabolite.

166. Which gene inactivation starts unchecked cell division at the crypt surface in colonic
adenomas?
A. FAP
B. APC
C. HNPCC
D. C-myc

Correct answer is B. RATIONALE: Carcinomas are found in as many as 4% of neoplastic polyps. Cells must accumulate 4-5 molecular defects, including activation of oncogenes and inactivation of tumor suppressor genes, to undergo malignant transformation. In normal mucosa, the surface epithelium regenerates approximately every 6 days. Crypt cells migrate from the base of the crypt to the surface, where they undergo differentiation, maturation, and, ultimately, lose the ability to replicate. In adenomas, several genetic mutations alter this process, starting with inactivation of the adenomatous polyposis coli (APC) gene, allowing unchecked cellular replication at the crypt surface. With the increase in cell division, further mutations occur, resulting in activation of the K-ras oncogene in the early stages and p53 mutations in the later stages.

173. Involvement of which of the following vessels is LEAST likely to result in a
margin-negative resection of a pancreatic cancer?
A. Portal vein
B. Superior mesenteric vein
C. Superior mesenteric artery
D. Splenic artery

Correct answer is C. RATIONALE: Partial involvement of the superior mesenteric vein (SMV) or portal vein makes this a borderline resectable tumor. The spleen can be removed. Involvement of the superior mesenteric artery (SMA) renders a pancreatic tumor unresectable.

194. What is the most common type of small bowel cancer?
A. Sarcoma
B. Lymphoma
C. Adenocarcinoma
D. Small cell carcinoma

Correct answer is C. RATIONALE: Adenocarcinoma is the most common histologic type of small bowel malignancy.

202. Which of the following treatments is most appropriate for a 9-cm unresectable
hepatocellular carcinoma that involves the portal vein?
A. Radiofrequency ablation
B. Transarterial chemoembolization (TACE)
C. External-beam radiation therapy
D. Percutaneous ethanol injection

Correct answer is C. RATIONALE: Radiofrequency ablation and stereotactic radiation therapy are good options for smaller lesions (usually less than 5 cm in diameter). TACE is contraindicated when the portal vein is involved.

208. Stage T2 adenocarcinoma of the gallbladder is found during a laparoscopic
cholecystectomy. What is the next most appropriate step in management?
A. Radical cholecystectomy and nodal dissection
B. Postoperative radiation therapy alone
C. Postoperative chemotherapy alone
D. Postoperative chemoradiation

Correct answer is A. RATIONALE: Radical resection is the only potentially curative treatment. Therefore, completion of lymphadenectomy should be performed after a simple cholecystectomy, except for the most superficial T1 tumors. While whole-liver tolerance is low (~30 Gy), one third of the liver can be treated to doses as high as 90 Gy. Liver tolerance is usually not a limiting factor in the adjuvant treatment of gallbladder cancer.

217. What is the incidence of adenocarcinoma and squamous cell carcinoma in U.S. patients with esophageal cancer?
Adenocarcinoma Squamous cell carcinoma
A. Increasing Increasing
B. Increasing Decreasing
C. Decreasing Decreasing
D. Decreasing Increasing

Correct answer is B. References: AU Bollschweiler E, Wolfgarten E, Gutschow C, Holscher AH SO. TI Demographic variations in the rising incidence of esophageal adenocarcinoma in white males. Cancer. Aug 1, 2001;92(3):549-55. AU Blot WJ, Devesa SS, Kneller RW, Fraumeni JF, Jr SO. TI Rising incidence of adenocarcinoma of the esophagus and gastric cardia. Journal of the American Medical Association. Mar. 13, 1991;265(10):1287-9. AU Blot WJ, Devesa SS, Kneller RW, Fraumeni JF, Jr SO. TI Rising incidence of adenocarcinoma of the esophagus and gastric cardia. Journal of the American Medical Association. Mar 13, 1991;265(10):1287-9. AU Blot. TI Esophageal cancer trends and risk factors. World Journal of Surgical Oncology, Seminars in Oncology, Aug 1994;21(4):403-10.

253. How many lymph nodes are required for accurate pathologic assessment after rectal cancer surgery?
A. 07
B. 12
C. 14
D. 17

Correct answer is C. Reference: Tepper JE. Journal of Clinical Oncology. January 1, 2001;Vol 19,No 1:pp 157-163.

276. Which of the following preoperative chemoradiation parameters is NOT superior to
postoperative chemoradiation in patients with rectal cancer?
A. Local control
B. Treatment compliance
C. Grade 3 or 4 toxicities
D. Overall survival

Correct answer is D. Reference: Sauer, et al. New England Journal of Medicine. 2004;351:1.

308. Which of the following statements about ulcerative colitis and colorectal cancer is true?
A. The risk for developing cancer is 30% in patients who have dysplasia.
B. The surveillance regimen is different than that used for Crohn's colitis.
C. Patients should have a colonoscopy to check for dysplastic changes every 5 years.
D. After 10 years, the incidence of colorectal cancer in patients with ulcerative colitis is
approximately 10% per year.

Correct answer is A. RATIONALE: The incidence of malignancy increases with duration. After 10 years, the incidence of colorectal cancer in patients with ulcerative colitis is approximately 1% per year.

232. Which of the following is TRUE regarding surgery for pancreatic cancer?
(A) A biopsy is required before a Whipple procedure.
(B) A Whipple procedure is performed for pancreatic tail and body lesions.
(C) Tumors involving the superior mesenteric vein may still be resectable.
(D) Tumor involvement of the portal vein is an absolute contraindication to surgery.

Key: C
Rationale: A biopsy is not required prior to performing a Whipple to remove a suspicious pancreatic tumor. Tumors with less than 180 degrees of involvement of the superior mesenteric artery may still be resectable. A Whipple procedure is performed primarily for tumors that arise in the pancreatic head. Portal vein resection with reconstruction may be performed for portal vein involvement by tumor.
References: Katz et al., J Am Coll Surg. 2008 May; 206(5):833-46.

316. The primary pattern of lymphatic spread for gallbladder cancer is to the:
A. porta hepatis.
B. retropancreaticoduodenal lymph nodes.
C. hepatoduodenal ligament to the celiac trunk.
D. aortocaval lymph nodes.

Correct answer is B. RATIONALE: The main route of lymphatic drainage from the gallbladder is to the cystic duct and downwards along the common duct to the retropancreaticoduodenal lymph nodes. Knowledge of the lymphatic drainage is important in understanding surgical procedures, their limitations, and radiation therapy planning.

324. The addition of mitomycin C to 5-fluorouracil (5-FU) and radiation therapy in patients with anal cancer improves all of the following EXCEPT:
A. local control.
B. colostomy-free survival.
C. disease-free survival.
D. overall survival.

Correct answer is D. Reference: Flam, et al. Journal of Clinical Oncology. 1996;14:2527-2539.

336. Which of the following TNM stages describes a rectal tumor that invades the submucosa with metastasis to four perirectal lymph nodes?
A. T1N1M0
B. T1N2M0
C. T1N3M0
D. T2N2M0

Correct answer is B. Reference: AJCC Cancer Staging Manual, 6th edition.

93. A phase III randomized study of preoperative chemotherapy versus chemoradiation for locally advanced GE junction cancers (German Esophageal Cancer Group) demonstrated that:
(A) the chemoradiation arm had a pathologic complete response rate of 25%.
(B) the chemotherapy arm had a pathologic complete response rate of 10%.
(C) there was a trend towards improved overall survival with chemotherapy alone.
(D) there was no significant difference in postoperative mortality.

Key: D
Rationale: This study randomized patients to receive induction chemotherapy followed by surgery or induction chemotherapy, followed by chemoradiation, then surgery. Patients treated with chemoradiation had a higher pathologic complete response rate (15.6% vs. 2%) and a trend towards improved overall survival (3 year survival 28% vs. 47%, p=0.07). There was a non-significant increase in postoperative mortality (10% vs. 3.8%, p=0.26) among patients treated with chemoradiation.
References: Stahl et al. JCO 27(6): 851-6, 2009.

3. Which of the following statements about microsatellite instability in colorectal cancer is FALSE?
A. It accounts for about 15% of sporadic colorectal cancers and most hereditary nonpolyposis cancers.
B. It is associated with a mucinous histology and a surrounding lymphoid reaction.
C. These tumors tend to be more prevalent in the proximal colon.
D. These tumors are associated with worse survival than stage-matched microsatellite stable tumors.

Correct answer is D. REFERENCES: Niv Y. Microsatellite instability and MLH1 promoter hypermethylation in colorectal cancer. World Journal of Gastroenterology. Mar 28, 2007;13(12):1767-9. Lynch HT, de la Chapelle A. Genetic susceptibility to nonpolyposis colorectal cancer. Journal of Medical Genetics. Nov 1999;36(11)801-18.

11. Which of the following trials demonstrated an improvement in survival for patients receiving preoperative radiation therapy for rectal cancer?
A. German Rectal Cancer Study Group
B. Swedish Rectal Cancer
C. Dutch CKVO
D. NSABP R-01

Correct answer is B. RATIONALE: The only randomized trial that has shown a survival benefit for preoperative radiotherapy was the Swedish Trial. The German and Dutch trials did not show a survival benefit. NSABP R-01 evaluated postoperative treatment.

20. Which of the following statements about adenocarcinoma of the anus is FALSE?
A. It typically arises within the anal mucosa or anal fistulas.
B. It represents approximately 5% to 10% of all cases of anal cancer.
C. It presents with more advanced disease than epidermoid carcinomas of the anal canal.
D. Definitive chemoradiation results in poor local control and high distant failure rates.

Correct answer: ABCD. REFERENCES: Papagikos M, Crane CH, Skibber J, Janjan NA, Feig B, Rodriguez-Bigas MA, Hung A, Wolff RA, Delclos M, Lin E, Cleary K. Chemoradiation for adenocarcinoma of the anus. International Journal of Radiation Oncology, Biology, Physics (Int J Radiat Oncol Biol Phys). Mar 1, 2003;55(3):669-78. Basik M, Rodriguez-Bigas MA, Penetrante R, Petrelli NJ. Prognosis and recurrence patterns of anal adenocarcinoma. American Journal of Surgery. Feb 1995;169(2):233-7.

28. Which of the following statements does NOT correctly describe a characteristic of HNPCC?
A. It is also known as Lynch syndrome.
B. It is the most common hereditary colorectal cancer syndrome.
C. Current diagnostic criteria include the Amsterdam criteria.
D. Promoter hypomethylation accounts for some cases of microsatellite instability.

Correct answer is D. REFERENCES: Hendriks YM, de Jong AE, Morreau H, Tops CM, Vasen HF, Wijnen JT, Breuning MH, Bröcker-Vriends AH. Diagnostic approach and management of Lynch syndrome (hereditary nonpolyposis colorectal carcinoma): a guide for clinicians. CA: A Cancer Journal for Clinicians (CA: Cancer J Clin). July-August 2006;56(4):213-25.

39. Which of the following statements about the extent of lymph node dissection in patients with gastric cancer is true?
A. A D2 dissection includes a splenectomy.
B. A D2 dissection did not improve survival when compared to a D1 dissection, according to a Dutch study.
C. The majority of patients received a D1 dissection in the INT-0116 randomized study evaluating postoperative chemoradiation.
D. Chemoradiation should not be given to patients after a D2 dissection.

Correct answer is B. RATIONALE: A splenectomy is associated with higher complications. The Dutch study did not show a survival benefit when comparing D1 vs. D2 dissections. The majority of patients in the INT-0116 study had a D0 dissection. One large study from Korea suggests a 5% overall survival benefit with chemoradiation, even with a D2 dissection. REFERENCE: Journal of Clinical Oncology (JCO) 22:2069-2077. International Journal of Radiation Oncology, Biology, Physics (Int. J. Radiation Oncology Biol. Phys.). 2005;63(5):1279–1285.

76. Which of the following outcomes was demonstrated in the INT-0123 (Minsky) study comparing the use of 50.4 and 64.8 Gy of radiation therapy with chemotherapy for esophageal cancer?
A. High-dose radiation therapy resulted in a significant survival benefit in the as-treated analysis, but outcomes were not improved in the high-dose group by intent-to-treat.
B. The benefit of high-dose radiation therapy was obscured due to patient deaths before a total dose of 50.4 Gy was reached.
C. There was no suggestion of a survival benefit with high-dose radiation therapy either in intent-to-treat or as-treated analysis.
D. Local control was significantly better in the high-dose radiation therapy arm.

Correct answer is C. RATIONALE: In the report of INT-0123, an as-treated analysis was performed due to the high incidence of early grade 5 toxicity in the high-dose radiation therapy arm. However, there remained no survival difference in the two arms. Local failure was not significantly different.

81. What is the AJCC stage for a patient with a tumor invading into, but not through, the muscularis propria with four positive lymph nodes after preoperative chemoradiation followed by resection?
A. pT2N2
B. pT3N2
C. ypT2N2
D. ypT3N2

Correct answer is C. RATIONALE: The "y" prefix in staging criteria is used for patients undergoing pretreatment. A tumor invading into but not through muscularis propria is classified as stage T2. Four or more lymph nodes would be classified as stage N2.

126. Which of the following is TRUE of pancreatic cancer?
(A) Most arise in the body/tail.
(B) It is associated with BRCA1.
(C) CEA is the most useful serum tumor marker.
(D) It is the tenth most common cancer diagnosed in the United States.

Key: D
Rationale: The 2011 cancer statistics show that pancreas cancer is the 10 most common cancer diagnosed and 4th deadliest cancer in the United States. Most pancreatic cancers arise in the pancreatic head. CA 19-9 is the most useful tumor marker of diagnosis and surveillance.
References: Siegel et al., CA Cancer J Clin. 2012 Jan-Feb; 62(1):10-29. Lillis-Hearne et al., Cancer of the Pancreas. In: Hoppe RT, Roach M, Phillips T, eds. Textbook of Radiation Oncology, 3rd edition, Elsevier, Philadelphia, PA; 2010: 801-819.

96. What is the most common histologic subtype of cholangiocarcinoma?
A. Adenocarcinoma
B. Neuroendocrine carcinoma
C. Transitional cell carcinoma
D. Squamous cell carcinoma

Correct answer is A. RATIONALE: More than 90% of cholangiocarcinomas are adenocarcinomas.

122. According to the CONKO (Oettle) trial, which of the following outcomes was associated with adjuvant therapy for patients with resected pancreatic cancer?
A. Local control was improved in patients receiving gemcitabine.
B. Local control was improved in patients receiving radiation therapy.
C. Disease-free survival was improved in patients receiving gemcitabine.
D. Disease-free survival was not affected by radiation therapy.

Correct answer is C. RATIONALE: This is an important trial showing the efficacy of chemotherapy in the adjuvant setting. Local control was not substantially different. Radiation was not used in this study.

135. According to the INT-0116 (Macdonald) study, which of the following was associated with postoperative chemoradiation and chemotherapy (3 cycles) versus surgery alone for resected gastric cancer?
A. Control of metastatic disease was improved with adjuvant therapy, leading to improved overall survival.
B. Locoregional control was improved with adjuvant therapy, leading to improved overall survival.
C. An increase in deaths was associated with adjuvant therapy, eliminating any survival benefit.
D. The study used cisplatin/5-fluorouracil (5-FU) chemotherapy.

Correct answer is B. RATIONALE: The INT-0116 study showed a survival benefit directly related to locoregional control. No difference in metastatic rate was seen. The study was positive by intent-to-treat. The backbone of chemotherapy was 5-fluorouracil/leucovorin (5-FU/LV). REFERENCE: New England Journal of Medicine (NEJM). 2001.

166. Which of the following genetic alterations is present in the majority of patients with pancreatic ductal adenocarcinomas?
A. MDM2 amplification
B. K-ras gene mutation
C. Platelet-derived growth factor receptor mutation
D. Deletion of the PTEN tumor suppressor gene

Correct answer is B. RATIONALE: Targeted-based therapies are becoming a critical component of oncology therapy, and in some sites, such as the colon, the mutation status of K-ras is predictive of a therapeutic response.

174. According to the German Rectal Cancer Study Group (Sauer) trial, what was the pathologic complete response rate to preoperative chemoradiation for patients who have rectal cancer?
A. 08%
B. 18%
C. 28%
D. 38%

Correct answer is A. RATIONALE: The German trial is one of the most important recent studies on rectal cancer. The pathologic complete response (CR) rate was reported. REFERENCE: Sauer, et al. New England Journal of Medicine (NEJM). 2004;351:1731-40.

189. Which of the following gene mutations has been shown to predict a patient's response to cetuximab therapy when used as second-line treatment of metastatic colorectal cancer?
A. Akt
B. K-ras
C. EGFR
D. VEGF

Correct answer is B. REFERENCE: Lièvre A, Bachet JB, Le Corre D, Boige V, Landi B, Emile JF, Côté JF, Tomasic G, Penna C, Ducreux M, Rougier P, Penault-Llorca F, Laurent-Puig P. K-ras mutation status is predictive of response to cetuximab therapy in colorectal cancer. Cancer Research. Apr 15, 2006;66(8):3992-5.

201. Which of the following statements about the incidence of esophageal cancer in the United States is true?
A. Squamous cell carcinomas are becoming more common.
B. There has been a six-fold increase in esophageal adenocarcinoma over the last 25 years.
C. The incidence of esophageal cancer is decreasing due to screening programs for Barrett's disease.
D. The proximal esophagus is the most common location of esophageal cancer.

Correct answer is B. REFERENCE: Pohl H and Welch HG. The role of over diagnosis and reclassification in the marked increase of esophageal adenocarcinoma incidence. Journal of National Cancer Institute. January 19, 2005;97(2):142-6.

223. Which of the following immunohistochemical staining patterns differentiates primary bronchogenic carcinoma from metastatic adenocarcinoma of gastrointestinal origin?
A. The expression of CDX2 is highly specific for pulmonary adenocarcinoma.
B. The expression of TTF-1 is important in distinguishing primary bronchogenic carcinoma from metastatic adenocarcinoma.
C. The expression of CK7− and CK20+ usually is associated with pulmonary adenocarcinoma.
D. The expression of CK7+ and CK20− usually is associated with metastatic adenocarcinoma of the colorectum.

Correct answer is B. RATIONALE: It is important to know some principles of lung cancer pathology. CDX2 is highly specific for gastrointestinal malignancies, which could help distinguish from primary lung adenocarcinoma. The majority of primary lung carcinomas are positive for TTF-1, whereas metastatic adenocarcinoma to the lung is virtually always negative. Primary lung adenocarcinoma is usually CK7+ and CK20-, therefore distinguishable from CK7- and CK20+ metastatic colorectal adenocarcinoma.

239. An increased risk of hepatocellular carcinoma is associated with exposure to:
A. hepatitis A.
B. Helicobacter pylori.
C. human papillomavirus.
D. aflatoxin B1.

Correct answer is D. RATIONALE: Aflatoxin B1 has been associated with an increased risk of hepatocellular carcinoma (HCC). Hepatitis B and C (not hepatitis A) are associated with an increased risk of HCC.

278. Which of the following statements about the MAGIC (Cunningham) study comparing perioperative ECF chemotherapy to surgery alone for resected gastric cancer is true?
A. A lower rate of curative surgery was demonstrated in patients who underwent perioperative chemotherapy.
B. A higher rate of postoperative complications was found in patients who underwent perioperative chemotherapy.
C. The overall and progression-free survival rate was improved in patients who underwent perioperative chemotherapy.
D. Most patients completed all 6 cycles of chemotherapy.

Correct answer is C. RATIONALE: The MAGIC study showed a higher rate of curative surgery with chemotherapy, a similar postoperative complication rate, and improved overall survival (OS) rate and progression-free survival (PFS) rate. However, only 41% of patients completed all chemotherapy. REFERENCE: Cunningham, et al. New England Journal of Medicine (NEJM). 2006.

294. Which of the following conditions is a contraindication for performing a radical resection on a patient with adenocarcinoma of the pancreas?
A. Tumor invasion of the splenic artery
B. Tumor encasement of the superior mesenteric artery
C. Involvement of the peripancreatic lymph nodes
D. Extensive pancreatic intraepithelial neoplasia

Correct answer is B. RATIONALE: Encasement of the superior mesenteric artery (SMA) is a contraindication of a Whipple procedure and is of major importance in pretreatment planning (i.e., definitive chemoradiation versus surgery).

303. Which of the following histologies is most commonly associated with small bowel cancer?
A. Carcinoid
B. Sarcoma
C. Lymphoma
D. Adenocarcinoma

Correct answer is D. RATIONALE: Adenocarcinoma — 45%; carcinoid — 29%; lymphoma — 16%; sarcoma — 10%. REFERENCE: Journal of National Cancer Institute. April 1987;78(4):653-6.

307. Which of the following statements about cholangiocarcinomas is true?
A. Surgery provides curative results in the majority of patients.
B. The majority of patients present with metastatic disease.
C. Proximal lesions have a worse prognosis than distal lesions.
D. Regional lymph nodes are rarely involved.

Correct answer is C. RATIONALE: Surgery, although having poor outcomes, is the mainstay of treatment for cholangiocarcinoma. Distal lesions have better outcomes than proximal tumors.

312. Which of the following tumor markers is commonly elevated in patients with hepatocellular carcinoma, but is rarely elevated in patients with cholangiocarcinoma?
A. CEA
B. CA 19-9
C. AFP
D. β-hCG

Correct answer is C. RATIONALE: α-Fetoprotein (AFP) is rarely positive in cholangiocarcinoma but is found in 35% to 75% of hepatomas. This may help distinguish between the two conditions during the initial workup.

326. Which of the following risk factors has NOT been shown to be associated with the development of anal cancer?
A. HIV infection
B. HPV infection
C. HSV infection
D. Multiple sexual partners

Correct answer is C. REFERENCE: Ryan DP, Compton CC, Mayer RJ. Carcinoma of the anal canal. New England Journal of Medicine (NEJM). March 16, 2000;342(11):792-800.

344. According to the GITSG 9173 trial, which of the following outcomes was associated with patients who had resected pancreatic cancer?
A. The median overall survival improved from 11 months to 20 months for patients in the chemoradiation arm.
B. The number of distant metastasis was decreased for patients in the chemoradiation arm versus the observation arm after surgery.
C. The same rate of survival was found in patients with positive margins as in those with negative margins.
D. A radiation dose of 60 Gy was superior to 40 Gy.

Correct answer is A. RATIONALE: This trial established the precedent of treating resected pancreatic cancer with chemoradiation.

356. According to the German Rectal Cancer Study Group (Sauer) trial, patients who received preoperative chemoradiation experienced which of the following outcomes in comparison to those treated with postoperative chemoradiation?
A. Increased disease-free survival
B. Increased acute toxicity
C. Decreased local recurrence
D. Decreased overall survival

Correct answer is C. RATIONALE: In the German trial, patients treated with preoperative chemoradiation had lower rates of locoregional recurrence and acute toxicity, with no difference in disease-free survival rates, compared to those treated with postoperative chemoradiation. REFERENCE: Sauer, et al. New England Journal of Medicine (NEJM), 2004;351:1731-40.

5. Which of the following drugs was shown in a phase III trial to improve survival in patients
with symptomatic locally advanced or metastatic pancreatic cancer?
A. Gemcitabine
B. Capecitabine
C. Bevacizumab
D. Cisplatin

Correct answer is A. REFERENCE: Burris, et al. Pivotal trial on gemcitabine for pancreatic
cancer. Journal of Clinical Oncology (JCO). 1997.

34. Which of the following characteristics is associated with the ESPAC-1 trial evaluating
treatment for patients with pancreatic cancer?
A. The trial was a two-arm randomized controlled trial.
B. Patients with resected ampullary cancer and resected pancreatic cancer were
included in the trial.
C. Overall survival was improved in patients receiving adjuvant chemoradiation.
D. Overall survival was improved in patients receiving adjuvant chemotherapy.

Correct answer is D. RATIONALE: Option A is incorrect, because the trial had a 2X2 factorial
design, but was really 3 separate trials. Option B is incorrect because only pancreatic cancer
was included. The EORTC trial included both. Option C is incorrect because survival was
worse in the chemoradiation group. Option D is correct because adjuvant chemoradiation
improved survival. REFERENCE: Neoptolemos, et al. New England Journal of Medicine
(NEJM). 2004.

49. A patient with liver fluke infestation has a higher risk for developing:
A. hepatoblastoma.
B. hepatocellular carcinoma.
C. fibrolamellar carcinoma.
D. cholangiocarcinoma.

Correct answer is D. RATIONALE: Liver fluke infestation has been associated with the
development of cholangiocarcinoma.

56. Which of the following treatments has improved overall survival in patients who have
advanced hepatocellular carcinoma with Child’s A cirrhosis?
A. Gemcitabine
B. Bevacizumab
C. 5-Fluorouracil
D. Sorafenib

Correct answer is D. RATIONALE: BACKGROUND: No effective systemic therapy exists for
patients with advanced hepatocellular carcinoma. A preliminary study suggested that sorafenib,
an oral multikinase inhibitor of the vascular endothelial growth factor receptor, the plateletderived
growth factor receptor, and Raf may be effective in hepatocellular carcinoma.
METHODS: In this multicenter, phase 3, double-blind, placebo-controlled trial, we randomly
assigned 602 patients with advanced hepatocellular carcinoma who had not received previous
systemic treatment to receive either sorafenib (at a dose of 400 mg twice daily) or a placebo.
Primary outcomes were overall survival and the time to symptomatic progression. Secondary
outcomes included the time to radiologic progression and safety. RESULTS: At the second
planned interim analysis, 321 deaths had occurred, and the study was stopped. Median overall
survival was 10.7 months in the sorafenib group and 7.9 months in the placebo group (hazard
ratio in the sorafenib group, 0.69; 95% confidence interval, 0.55 to 0.87; P<0.001). There was
no significant difference between the two groups in the median time to symptomatic progression
(4.1 months vs. 4.9 months, respectively, P=0.77). The median time to radiologic progression
was 5.5 months in the sorafenib group and 2.8 months in the placebo group (P<0.001). Seven
patients in the sorafenib group (2%) and two patients in the placebo group (1%) had a partial
response; no patients had a complete response. Diarrhea, weight loss, hand-foot skin reaction,
and hypophosphatemia were more frequent in the sorafenib group. CONCLUSIONS: In patients
with advanced hepatocellular carcinoma, median survival and the time to radiologic progression
were nearly 3 months longer for patients treated with sorafenib than for those given a placebo.
REFERENCES: (ClinicalTrials.gov number, NCT00105443.) AD Barcelona Clinic Liver Cancer
Group, Institut d'Investigacions Biomediques August Pi i Sunyer, Centro de Investigaciones en
Red de Enfermedades Hepaticas y Digestivas Hospital Clinic Barcelona, Barcelona.
jmllovet@clinic.ub.es. Sorafenib in advanced hepatocellular carcinoma. AULlovet JM; Ricci S;
Mazzaferro V; Hilgard P; Gane E; Blanc JF; de Oliveira AC; Santoro A; Raoul JL; Forner A;
Schwartz M; Porta C; Zeuzem S; Bolondi L; Greten TF; Galle PR; Seitz JF; Borbath I;
Haussinger D; Giannaris T; Shan M; Moscovici M; Voliotis D; Bruix J SON, New England
Journal of Medicine (NEJM). July 24, 2008;359(4):378-90.

64. Which of the following complications is LEAST likely to occur after a total gastrectomy?
A. Slower intestinal passage due to loss of reservoir function
B. Increased susceptibility to enteric infections due to loss of bacteriostatic function
C. Eventual development of pernicious anemia due to loss of erythropoietic function
(intrinsic factor)
D. Demineralization of bones due to impaired calcium absorption

Correct answer is A. RATIONALE: Dumping syndrome with loss of reservoir function needs to
be recognized after a patient has had a total gastrectomy, and the patient should be
encouraged to consume frequent small meals. REFERENCE:
http://www.netterimages.com/images/vpv/000/000/006/6684-0550x0475.jpg

71. A biopsy of an enhancing liver mass demonstrates adenocarcinoma, with positive
staining for CK7 and CK20 but negative results for TTF-1. These findings are most
consistent with:
A. metastatic colorectal cancer.
B. metastatic non-small cell lung cancer.
C. primary intrahepatic cholangiocarcinoma.
D. hepatocellular carcinoma.

Correct answer is C. RATIONALE:
CK7+ CK20+
Urothelial tumors
Mucinous ovarian cancer
Pancreatic or biliary
cancer
CK7+ CK20-
Non-small cell lung cancer
Small cell lung cancer
Breast cancer
Endometrial cancer
Nonmucinous ovarian
cancer
Mesothelioma
Squamous cancer of
cervix
CK7- CK20+
Colorectal
cancer
Merkel cell
cancer
CK7- CK20-
Hepatocellular cancer
Renal cell cancer
Prostate cancer
Squamous cell lung
cancer
Head and neck cancer

83. Which of the following statements about the treatment of gastric cancer is true?
A. A D2 lymph node dissection is defined by the removal of more than six lymph
nodes.
B. In the INT-0116 randomized intergroup (Macdonald) trial, adjuvant therapy improved
locoregional control and overall survival.
C. In the MAGIC (Cunningham) trial, perioperative chemotherapy improved the
pathologic complete response rate.
D. In the randomized Dutch Gastric Cancer trial, D2 versus D1 lymphadenectomy
increased overall survival.

Correct answer is B. RATIONALE: D2 lymphadenectomy is not defined by the number of
lymph nodes removed (TNM stage N2 refers to 7-15 lymph nodes being positive, not to be
confused with D2). Perioperative chemotherapy did not improve pathological complete
response (CR) rates. D2 lymphadenectomy did not improve survival over D1 lymphadenectomy
for patients with gastric cancer.

84. According to randomized clinical trial data, which of the following statements about
chemoradiation for esophageal cancer is true?
A. Most of the patients in the RTOG 85-01 trial had esophageal squamous cell
carcinomas.
B. The CALGB 9781 trial established the equivalence of chemoradiation to surgery for
resectable tumors.
C. In the randomized German Esophageal Cancer Study Group (Stahl) trial, high-dose
chemoradiation had a lower local failure rate than standard-dose chemoradiation,
followed by surgery.
D. The INT 0123 (Minsky) trial showed that increasing the radiation dose for definitive
chemoradiation improved survival.

Correct answer is A. RATIONALE: Know current randomized clinical trial data (RTOG 85-01
trial, CALGB 9781 trial, German Esophageal Cancer Study Group (Stahl) trial, and INT 0123
(Minsky) trial data).

94. Based on a secondary analysis of the EORTC 22921 Rectal Study, adjuvant
chemotherapy benefited patients who had rectal cancer and:
A. tumors >3 cm on pathologic assessment.
B. required an abdominal-perineal resection.
C. stage ypT0-T2.
D. stage ypN+.

Correct answer is C. REFERENCE: Journal of Clinical Oncology. 25:4379-4386.

104. Which of the following statements about small bowel tumors is FALSE?
A. Most small bowel tumors occur in the duodenum or proximal jejunum.
B. Most GI carcinoid tumors are located in the appendix, small bowel, or rectum.
C. Patients with Crohn's disease and Peutz-Jeghers syndrome have a higher incidence
of small bowel malignancy.
D. Patients with primary small bowel lymphomas are more likely to have Hodgkin
lymphoma versus non-Hodgkin lymphoma.

Correct answer is D. REFERENCE: Schottenfeld D, Beebe-Dimmer JL, Vigneau FD. The
epidemiology and pathogenesis of neoplasia in the small intestine. Annals of Epidemiology (Ann
Epidemiol). Jan. 2009;19(1):58-69.

122. Regarding extrahepatic cholangiocarcinoma,:
(A) papillary subtype carries the worst prognosis.
(B) Klatskin tumors arise near the Ampulla of Vater.
(C) primary sclerosing cholangitis is a risk factor.
(D) gemcitabine alone is the optimal chemotherapy.

Key: C
Rationale: In the randomized ABC trial, gemcitabine/cisplatin was shown to have superior survival over gemcitabine alone. The papillary subtype carries the best prognosis. Klatskin tumors arise at the bifurcation of the right and left hepatic ducts. PSC carries a 10-15% risk of developing cholangiocarcinomas.
References: Valle et al., N Engl J Med. 2010 Apr 8; 362(14):1273-81. Jarnagin Ann Surg. 2005 May; 241(5):703-12. Bergquist et al, J Hepatol. 2002 Mar; 36(3):321-7.

109. Based on the German Rectal Cancer Study, which of the following outcomes in patients
with rectal cancer was significantly improved by preoperative chemoradiation versus
postoperative chemoradiation?
A. Pelvic control
B. Overall survival
C. Distant metastatic rate
D. Postoperative wound complications

Correct answer is A. RATIONALE: Preoperative chemoradiotherapy, as compared with
postoperative chemoradiotherapy, improved local pelvic control in patients with rectal cancer. It
also was associated with reduced toxicity, but it did not improve overall survival. REFERENCE:
New England Journal of Medicine (NEJM). 2004;351:1731-40.

117. Chemoradiation followed by surgery for esophageal cancer is LEAST likely to produce:
A. higher local relapse rates than definitive chemoradiation.
B. greater pathological complete response rates than chemotherapy alone.
C. improved overall survival compared to surgery alone, based on the Walsh study.
D. pathological complete response rates that are correlated with overall survival.

Correct answer is A. RATIONALE: Adding surgery to chemoradiation reduces the local relapse
rate but does not impact overall survival (admittedly, this was for squamous cell carcinomas).
REFERENCE: Stahl M, Stuschke M, Lehmann N, Meyer HJ, Walz MK, Seeber S, Klump B,
Budach W, Teichmann R, Schmitt M, Schmitt G, Franke C, Wilke H. Chemoradiation with and
without surgery in patients with locally advanced squamous cell carcinoma of the esophagus.
Journal of Clinical Oncology (J Clin Oncol). Apr 1, 2005;23(10):2310-7.

124. According to the MRC CR07 randomized trial, a short course of preoperative radiation
therapy (5 Gy x 5 fractions) versus selective postoperative chemoradiation significantly
reduced the local recurrence rate for patients with rectal cancer in which of the following
locations?
A. Lower portion of the rectum
B. Higher portion of the rectum
C. Mid-portion of the rectum
D. All locations

Correct answer is D. RATIONALE: Preliminary results indicate that a routine short course of
preoperative radiation therapy significantly reduced local recurrence and improved disease-free
survival at 3 years when compared with a highly selective postoperative approach.
REFERENCE: Lancet. 2009;373:811–20.

127. A 50-year-old woman presents with a mass in the lesser curvature of the stomach, and
biopsy results reveal that she has MALT lymphoma. Which of the following treatment
options would be most appropriate?
A. Gastrectomy with D2 nodal dissection for a patient without Helicobacter pylori
infection
B. Local radiation therapy to 30 Gy for a patient with or without Helicobacter pylori
infection
C. Combined multiagent chemotherapy and involved-field radiation therapy to reduce
the risk that MALT lymphoma will transform into a more aggressive type of
lymphoma
D. First-line therapy for Helicobacter pylori infection and monitoring of the patient’s
treatment response for several months before considering use of involved-field
radiation therapy

Correct answer is D. RATIONALE: Patients with gastric MALT lymphoma and Helicobacter
pylori infection should receive first-line therapy for Helicobacter pylori infection, and their
treatment response should be evaluated for at least 6 months before making a decision on
whether to include second-line therapy of involved-field radiation therapy. REFERENCES:
Bayerdorffer E, Neubauer A, Rudolph B, et al. Regression of primary gastric lymphoma of
mucosa-associated lymphoid tissue type after cure of Helicobacter pylori infection. MALT
Lymphoma Study Group. Lancet. 1995;345:1591–1594. Roggero E, Zucca E, Pinotti G, et al.
Eradication of Helicobacter pylori infection in primary low-grade gastric lymphoma of mucosaassociated
lymphoid tissue. Annals of Internal Medicine (Ann Intern Med). 1995;122:767–769.

139. Based on the CALGB 8984 phase II study, what was the local recurrence rate at
10 years for patients with stage T2 rectal tumors after full-thickness wide local excision
with or without chemoradiation?
A. <10% with chemoradiation
B. <10% without chemoradiation
C. Approximately 20% with chemoradiation
D. Approximately 20% without chemoradiation

Correct answer is C. RATIONALE: In this study, all patients with stage T2 rectal cancer
disease received 54 Gy of irradiation with 5-FU postoperatively. In spite of this treatment
regimen, 10-year results showed a local recurrence rate of 18%. For this reason, fewer centers
offer wide local excision (WLE) for stage T2 rectal tumors. REFERENCE: Dis Colon Rectum.
Aug 2008;51(8):1185-91; discussion 1191-4.

154. Which of the following organs is an intraperitoneal structure?
A. Rectum
B. Ascending colon
C. Descending colon
D. Transverse colon

Correct answer is D. RATIONALE: The transverse colon is an intraperitoneal structure, while
the other three structures are retroperitoneal.

170. Which of the following characteristics is associated with the Intergroup 0130 randomized
trial that evaluated postoperative chemoradiation for patients with high-risk colon
cancer?
A. Only patients with stage T4 disease were eligible.
B. Patients with negative surgical margins were excluded.
C. Radiation therapy had no disease-free survival benefit.
D. Radiation therapy improved overall survival.

Correct answer is C. RATIONALE: The Intergroup 0130 was a randomized trial that evaluated
the addition of adjuvant radiation therapy to chemotherapy in patients with resected colon
cancer. Patients were eligible if they had stage T3N+ or T4 disease. The trial was closed due
to poor accrual, but there was no significant difference in either disease-free or overall survival.

185. Which of the following stages is most appropriate for a patient who has colon cancer that
invades the subserosa with five positive lymph nodes?
A. T2N1 (IIIA)
B. T3N1 (IIIB)
C. T3N2 (IIIC)
D. T4N2 (IIIC)

Correct answer is C. RATIONALE: A tumor that has invaded the subserosa is a stage T3
lesion, and involvement of four or more lymph nodes is stage N2 disease.

199. Which of the following treatments would be most appropriate for a patient who has anal
canal cancer, is HIV positive, and has a CD4 count of 800 cells/mm3?
A. Radiation therapy alone to a reduced field
B. Chemoradiation with 5-FU and mitomycin C
C. Chemoradiation with 5-FU alone
D. Chemotherapy alone

Correct answer is B. RATIONALE: Standard chemoradiation is an appropriate treatment for
anal canal cancer in a patient who is HIV positive and has an adequate CD4 count.

215. Which of the following conclusions about the addition of chemotherapy to radiation
therapy for anal cancer is supported by randomized trial data?
A. Mitomycin C/5-FU improves overall survival versus radiation therapy alone.
B. Mitomycin C/5-FU improves overall survival versus 5-FU alone.
C. Mitomycin C/5-FU decreases the colostomy rate versus 5-FU alone.
D. Cisplatin/5-FU improves colostomy-free survival versus mitomycin C/5-FU.

Correct answer is C. RATIONALE: The RTOG 8704 study compared concurrent radiation
therapy (RT) with 5-FU with or without mitomycin C. There was no significant difference in
overall survival, but there was a significant decrease in colostomy rates in the mitomycin C arm
(use of mitomycin C and 5-FU). Cisplatin-based chemotherapy was evaluated in RTOG 98-11,
which randomized patients to induction cisplatin/5-FU, followed by concurrent RT with
cisplatin/5-FU versus the standard concurrent 5-FU/ mitomycin C with RT. The cisplatin arm
had a significantly worse colostomy rate. Both the EORTC of the UKCCR showed an
improvement in colostomy rates with chemoradiation (5-FU/mitomycin C) versus RT alone, but
no significant difference in overall survival.

228. Which of the following conditions most likely would require a treatment break for a
patient receiving chemoradiation for anal canal cancer?
A. Moist desquamation
B. Secondary infection of desquamated skin
C. Two to three episodes of diarrhea daily
D. Rectal irritation during defecation

Correct answer is B. RATIONALE: Treatment breaks, while often necessary, should be limited
as much as possible when patients are receiving chemoradiation for anal canal cancer. Moist
desquamation in itself is not an indication for a break, unless there are signs of an infection
present. Rectal irritation should be managed conservatively. Diarrhea should only result in a
break if it cannot be controlled with medications.

257. Which of the following tumor stages is most appropriate for a 2-cm pancreatic
adenocarcinoma that involves the duodenum but does not involve the celiac axis or
superior mesenteric artery?
A. T1
B. T2
C. T3
D. T4

Correct answer is C. RATIONALE: Tumor that extends beyond the pancreas but without celiac
axis or superior mesenteric artery (SMA) involvement is stage T3.

271. Which of the following regions of tumor involvement is classified as stage T3 ampulla of
Vater adenocarcinoma?
A. Extension beyond the ampulla
B. Invasion of the pancreas
C. Invasion of the duodenum
D. Invasion of adjacent organs

Correct answer is B. RATIONALE: Ampullary adenocarcinoma that invades the pancreas is
regarded as stage T3.

2. According to the Intergroup 0123 trial, patients who received 64.8 Gy compared to
50.4 Gy for esophageal cancer had:
A. similar overall survival rates.
B. significantly higher overall survival rates.
C. significantly lower overall survival rates.
D. significantly improved local control.

Correct answer is A. RATIONALE: The Intergroup 0123 trial showed no significant difference
in survival between the two arms (64.8 Gy compared to 50.4 Gy) for patients with esophageal
cancer. Concurrent 5-FU and cisplatin was administered in both arms. REFERENCE: Minsky,
et al. Journal of Clinical Oncology (J Clin Oncol). 2002.

18. Which of the following factors most significantly increases the risk for the development of
esophageal adenocarcinoma?
A. Female gender
B. Barrett's esophagus
C. Alcohol consumption
D. Cigarette smoking

Correct answer is B. RATIONALE: The incidence of gastroesophageal junction
adenocarcinoma is increasing in the United States, especially among white males and females.
Barrett's esophagus is one of the most important risk factors for developing esophageal
adenocarcinoma. Both smoking and alcohol are risk factors for developing esophageal
squamous cell carcinoma.

47. According to the MOSAIC trial, which outcome was demonstrated in patients with
resected colon cancer?
A. Overall survival was improved with the addition of oxaliplatin to 5-FU and leucovorin
(FOLFOX) versus 5-FU and leucovorin alone.
B. Disease-free survival was improved with FOLFOX in patients with stage II and
stage III disease.
C. Metastatic disease was decreased with the addition of bevacizumab to 5-FU,
leucovorin, and oxaliplatin therapy.
D. Toxicity was decreased with FOLFOX versus 5-FU and leucovorin alone.

Correct answer is A. RATIONALE: The MOSAIC trial randomized more than 2000 patients
with stage II and stage III colon cancer after resection to 5-FU and leucovorin alone versus 5-FU
and leucovorin with the addition of oxaliplatin (FOLFOX). Overall survival (OS) and
disease-free survival (DFS) were significantly improved in the FOLFOX group. However, DFS
was significantly improved only in patients with stage III disease and not in patients with stage II
disease. Bevacizumab (Avastin) was not used in this trial. Toxicity was significantly increased
in the FOLFOX arm.


54. Which of the following genetic alterations has been shown to be predictive of a negative
response to the EGFR-inhibitor cetuximab in patients with metastatic colon cancer?
A. PTEN deletion
B. KRAS mutation
C. MDM2 amplification
D. TP53 loss of heterozygosity

Correct answer is B. RATIONALE: KRAS mutations have been shown to be predictive of a
lack of response to both cetuximab and panitumumab. Given that the benefit of these agents is
restricted to KRAS wild-type patients, ASCO recommendations now reflect that patients should
have KRAS testing prior to initiation of anti-EGFR therapies, and if they are KRAS-mutant,
alternative therapies should be considered.

86. A 4 cm anal canal tumor with no lymphadenopathy should be classified as stage:
A. T4N0.
B. T3N0.
C. T2N0.
D. T1N0.

Correct answer is C. RATIONALE: Anal canal tumors that are >2 cm but 5 cm are classified
as stage T2.

100. Which of the following management options is most appropriate for a patient who has
clinically residual anal cancer 4 weeks after completion of chemoradiation?
A. Referral for salvage chemotherapy
B. Referral for immediate salvage abdominoperineal resection
C. Continued close clinical observation until disease progression
D. Radiographic imaging to evaluate local disease progression

Correct answer is C. RATIONALE: Squamous cell carcinoma (SCC) of the anal canal can
regress slowly. Therefore, residual disease 4 weeks after therapy is not uncommon. Typically,
biopsy is reserved for clinical progression or lack of complete regression at a later point in time
(some advocate 12 weeks). If there is disease progression, a salvage abdominoperineal
resection (APR) is appropriate. Chemotherapy would be reserved for metastatic disease
recurrence. Patients are typically followed by a clinical exam and then a biopsy to document
disease progression or persistence. Imaging may be obtained prior to surgery to ensure that
there is no metastatic disease, but it would not typically be part of the surveillance protocol.

142. The phase III CONKO-001 trial evaluated the efficacy and toxicity of which adjuvant
treatment after complete resection of pancreatic cancer?
A. Gemcitabine versus observation
B. Gemcitabine and oxaliplatin versus gemcitabine
C. 5-FU-based chemoradiation with gemcitabine versus 5-FU
D. Preoperative versus postoperative chemotherapy

Correct answer is A. RATIONALE: CONKO-001 is a study that is often quoted in the medical
oncology literature to justify omission of adjuvant chemoradiation therapy for pancreatic cancer.
It is worth knowing that this study exists. Treatment with gemcitabine for 6 months after
complete resection of pancreatic cancer significantly increased overall survival compared with
observation alone (presented, not yet published) and delayed the development of recurrent
disease (published in JAMA, 2007).

152. Which of the following statements about the use of adjuvant radiation therapy for
resected colon cancer is true?
A. There has been no prospective trial examining the role of radiation therapy for these
patients.
B. The INT 0130 trial showed that chemoradiation improved survival in these patients.
C. The INT 0130 trial showed that chemoradiation improved local control in these
patients.
D. Retrospective data suggest that chemoradiation may be beneficial in patients with a
high risk of local recurrence.

Correct answer is D. RATIONALE: Retrospective data has suggested that the addition of
chemoradiotherapy in patients with a high risk of local recurrence (i.e., patients with stage
T3-T4, N+ disease) may decrease recurrence rates. The INT 0130 is a prospective randomized
trial that randomized 222 patients with resected colon cancer (T3N1-2 tumors or tumors with
adhesion or invasion into adjacent structures) to 5-FU/Levamisole with or without radiation
therapy. The trial closed prematurely due to poor accrual. There were no differences in any
end points; however, the small number of patients analyzed may limit the conclusions of this
study. Because of this, there are those who would advocate for postoperative chemoradiation
in high-risk patients in “fixed” areas of the colon – i.e., positive margins, perforations, fistulas.

156. Which of the following findings is associated with pancreatic cancer?
A. High (>75%) incidence of KRAS mutations
B. High (>75%) incidence of HER2/neu overexpression
C. Diagnosis confirmed by an elevated serum CA 19-9 level
D. Low incidence of positive lymph nodes

Correct answer is A. RATIONALE: A KRAS codon 12 mutation frequency of 70% to 100% is
the highest reported for any tumor type. Ca 19-9 serum levels may be elevated due to biliary
obstruction. Seventy to eighty percent of patients have node-positive disease.

170. Which of the following is most characteristic of gallbladder cancer?
A. Isolated locoregional failures are more common than in extrahepatic
cholangiocarcinoma.
B. Stage T2 disease involves invasion of the perimuscular connective tissue.
C. Peritoneal seeding is increased in cancers on the hepatic parenchymal surface than
on the serosal surface.
D. It is associated with hepatitis B and C virus infection.

Correct answer is B. RATIONALE: Gallbladder cancer is more likely to recur with distant
failures and less likely to recur with isolated locoregional failures compared to extrahepatic
cholangiocarcinoma. A study by Jarnagin, et al., found that of those patients who developed
disease recurrence, isolated locoregional disease as the first site of failure occurred in 15% of
patients with gallbladder cancer compared with 59% of patients with cholangiocarcinoma.
According to the 2009 AJCC 7th edition staging system, stage T2 disease is defined by invasion
into the perimuscular connective tissue without extension into the serosa or liver.
REFERENCES: Jarnagin, et al. Cancer. 98(8):1689-700. AJCC Cancer Staging Manual. 7th ed.
2009.

190. What was the 1-year local control rate after administration of individualized stereotactic
body radiation therapy for liver metastases with no size limits, according to a
multi-institutional phase I study?
A. 29%
B. 52%
C. 71%
D. 87%

Correct answer is C. RATIONALE: In this multi-institutional study of individualized stereotactic
body radiation therapy (SBRT) that used dosing based on liver metastases, the 1-year local
control rate was estimated to be 71%. It's important to note that there was no size restriction in
this study. REFERENCE: Lee MT, et al. Journal of Clinical Oncology (J Clin Oncol).
2009;27:1585-1591.


198. According to the SHARP trial, which outcome was significantly improved with sorafenib
versus a placebo in patients with advanced hepatocellular carcinoma?
A. Time to symptomatic progression
B. Time to radiographic progression
C. Grade 3 or greater GI toxicity
D. Response rate

Correct answer is B. RATIONALE: Overall survival and time to radiographic progression were
both improved. However, time to symptomatic progression was not improved. Single-agent
sorafenib was associated with a low (2.3%) response rate. Side effects include diarrhea and
hand-foot syndrome. REFERENCE: New England Journal of Medicine (N Engl J Med).
2008;359:378-90.

212. Which of the following factors is included in the Cancer of the Liver Italian Program
(CLIP) score for predicting survival in patients with hepatocellular carcinoma without
extrahepatic metastases?
A. CEA level
B. Weight loss
C. Portal vein thrombosis
D. Tumor grade

Correct answer is C. RATIONALE: Factors not used in determining the CLIP score include
CEA level, weight loss, or tumor grade. The CLIP score is important for radiation oncologists in
comparing results of liver-directed therapy. The following factors (illustrated in the table below)
are used to determine the CLIP score.
Child-Pugh stage
A 0
B 1
C 2
Tumor morphology
Uninodular and extension ≤50% 0
Multinodular and extension ≤50% 1
Massive or extension >50% 2
Alpha-fetoprotein
<400 0
≥400 1
Portal vein thrombosis
No 0
Yes 1

225. Which of the following management options would NOT be appropriate for a patient who
has negative lymph nodes and surgical margins after resection for extrahepatic
cholangiocarcinoma?
A. Observation
B. 5-FU-based chemoradiation
C. Gemcitabine-based chemotherapy
D. SBRT to the surgical bed

Correct answer is D. RATIONALE: Management of resected cholangiocarcinomas remains
controversial due to the lack of randomized data. However, according to NCCN guidelines,
observation, chemoradiation, and chemotherapy are reasonable treatment options.

238. According to the EORTC 22921 randomized trial, which outcome was significantly
improved by chemotherapy administered concurrently with preoperative radiation
therapy or as adjuvant therapy in patients with locally advanced rectal cancers?
A. Local control
B. Distant metastases-free survival
C. Overall survival
D. Late toxicity

Correct answer is A. RATIONALE: EORTC 22921 randomized 1011 patients with resectable
T3 or T4N0 rectal cancer by 2 x 2 randomization to either receive 1) preoperative radiation
therapy alone; 2) preoperative chemoradiotherapy alone; 3) preoperative radiation therapy and
postoperative chemotherapy; or 4) preoperative chemoradiotherapy and postoperative
chemotherapy. With a median follow-up of 5.4 years, there was no significant difference in
overall survival between the groups that received chemotherapy preoperatively or
postoperatively (p=0.12). However, patients that received chemotherapy either preoperatively
or postoperatively were found to have significantly lower rates of local recurrence compared to
patients who did not receive chemotherapy (p=0.002). Specifically, the 5-year cumulative
incidence rates of local recurrence were 8.7%, 9.6%, and 7.6% for patients treated with
chemotherapy preoperatively, postoperatively, and both, respectively compared to 17.1%
among patients treated with radiation therapy alone. The cumulative incidence of distant
metastases did not differ significantly according to the preoperative or the postoperative
treatment (p=0.14, p=0.62, respectively). There was no difference in late toxicity between the
four treatment groups. REFERENCE: Bosset JF, Collette L, Calais G, et al. Chemotherapy with
preoperative radiotherapy in rectal cancer. New England Journal of Medicine (N Engl J Med).
2006;355:1114-23.

251. According to the German Rectal Cancer Study, what was the rate of pathologic complete
response in patients receiving preoperative chemoradiotherapy?
A. 00%
B. 08%
C. 24%
D. 32%

Correct answer is B. RATIONALE: Between 1995 and 2002, the German Rectal Cancer Study
Group enrolled 823 patients in a phase III trial comparing conventionally fractionated
preoperative radiation therapy given with concurrent 5-FU chemotherapy (n=421) with the same
treatment given postoperatively (n=402) in patients with clinical stage T3/T4 or node-positive
rectal cancer between 1995 and 2002. Preoperative radiation therapy consisting of a total of
50.4 Gy given over 28 days (delivered in fractions of 1.8 Gy per day, five days per week) was
prescribed to the primary tumor and pelvic lymph nodes, and postoperative radiation therapy
included an additional boost of 5.4 Gy. Concurrent continuous infusion 5-FU was given over
5 days at a dose of 1000 mg/m2/day during the first and fifth weeks of radiation therapy, and all
patients received 4 additional cycles of adjuvant 5-FU (500 mg/m2/day). Surgery was performed
within 6 weeks after completion of chemoradiation. With a median follow-up of 45.8 months,
preoperative chemoradiation decreased 5-year cumulative incidence of pelvic relapse compared
to the postoperative treatment group (5-year LF: 6% vs. 13%, p=0.006). There was no
significant difference with respect to disease-free survival (5-year DFS: 68% vs. 65%, p=0.80)
or overall survival between the two groups (5-year OS: 76% vs. 74%, p=0.80). Additionally,
preoperative radiation therapy was significantly associated with tumor downstaging, yielding an
8% pathologic complete response rate and a 15% absolute reduction in lymph node
involvement (25% vs. 40%). REFERENCE: Sauer R, Becker H, Hohenberger W, et al.
Preoperative versus postoperative chemoradiotherapy for rectal cancer. New England Journal
of Medicine (N Engl J Med) 2004;351:1731-40.

264. What is the stage for a patient who has rectal adenocarcinoma with perirectal fat
invasion, two positive perirectal lymph nodes, and no distant metastases?
A. T2N1M0
B. T3N1M0
C. T3N2M0
D. T4N2M0

Correct answer is B. REFERENCE: AJCC Cancer Staging Manual. 7th ed. 2009.

276. According to the Swedish Rectal Cancer Trial, which treatment improved local control
and overall survival in patients with resectable rectal cancer?
A. Total mesorectal excision
B. Preoperative radiation therapy with continuous 5-FU infusion
C. Postoperative radiation therapy
D. Preoperative radiation therapy

Correct answer is D. RATIONALE: The Swedish Rectal Cancer Trial was the one of the first
randomized studies to evaluate the efficacy of this fractionation schema, which randomized
908 patients with stage I-III resectable rectal cancer between 1987 and 1990 to curative surgery
alone (n=454) or preoperative radiation therapy (25 Gy in 5 fractions, n=454) followed by
curative surgery within 1 week of completion of radiation. With a median follow-up of 13 years,
compared to surgery alone, preoperative radiation therapy significantly decreased rates of local
recurrence (9% vs. 26%, p<0.001) and increased rates of cancer-specific survival (72% vs.
62%, p=0.03) and overall survival (38% vs. 30%, p=0.008)1. However, there were no significant
differences with respect to rates of distant failure between the irradiated and nonirradiated arms.
REFERENCE: Improved survival with preoperative radiotherapy in resectable rectal cancer.
Swedish Rectal Cancer Trial. New England Journal of Medicine (N Engl J Med).
1997;336:9807.

288. In the Intergroup randomized (Macdonald, et al.) trial, patients with gastric cancer
received postoperative radiation therapy administered concurrently with:
A. 5-FU and cisplatin.
B. 5-FU and leucovorin.
C. 5-FU protracted venous infusion.
D. ECF chemotherapy.

Correct answer is B. REFERENCE: Macdonald, et al. Landmark trial on postoperative
chemoradiation for gastric cancer. New England Journal of Medicine (N Engl J Med). 2001.

295. In the Intergroup randomized (Macdonald, et al.) trial, what postoperative radiation dose
was administered concurrently with chemotherapy for patients with gastric cancer?
A. 30.6 Gy
B. 45.0 Gy
C. 50.4 Gy
D. 54.0 Gy

Correct answer is B. REFERENCE: Macdonald, et al. Landmark trial on postoperative
chemoradiation for gastric cancer. New England Journal of Medicine (N Engl J Med). 2001.

302. D2 lymphadenectomy for gastric cancer does NOT include dissection of the:
A. celiac axis lymph nodes.
B. perigastric lymph nodes.
C. paraaortic lymph nodes.
D. lymph nodes along the splenic vessels.

Correct answer is C. RATIONALE: D2 lymphadenectomy for gastric cancer includes dissection
of perigastric lymph nodes and nodes along the celiac axis, left gastric, hepatic, and splenic
vessels.

309. According to the MAGIC trial, patients administered perioperative chemotherapy for
gastric cancer:
A. received epirubicin, capecitabine, and 5-FU.
B. had equivalent overall survival to those patients having surgery alone.
C. had a diagnosis of squamous cell carcinoma.
D. included those with gastroesophageal junction tumors.


Correct answer is D. RATIONALE: Patients in the perioperative chemotherapy arm received
epirubicin, cisplatin, and 5-FU (ECF) chemotherapy. Patients in the perioperative
chemotherapy arm had significantly higher overall survival than those having surgery alone.
The trial included patients with adenocarcinomas only (not squamous cell carcinomas) and
those with gastroesophageal junction tumors. REFERENCE: Cunningham, et al. Landmark trial
on perioperative chemotherapy for gastric cancer. New England Journal of Medicine (New Engl
J Med). 2006.

1. Which of the followingrepresents primary tumors listed in order of increasing risk of distantmetastatic diseases following surgical resection?


(A) Intrahepaticcholangiocarcinoma, hilar cholangiocarcinoma, adenocarcinoma of the gallbladder


(B) Hilar cholangiocarcinoma,intrahepatic cholangiocarcinoma, adenocarcinoma of the gall bladder


(C) Adenocarcinoma of the gallbladder, hilar cholangiocarcinoma, intrahepatic cholangiocarcinoma


(D) Hilar cholangiocarcinoma,adenocarcinoma of the gall bladder, intrahepatic cholangiocarcinoma

Key: A Rationale: In the study by Jarnagin et al., the rate of firstrecurrence including a distant site for adenocarcinoma of the gall bladder was85% vs. 41% for patients with hilar cholangiocarcinoma. In the study by Weberet al., the rate of first recurrence for an intrahepatic cholangiocarcinomaincluding a distant site was 25%. References:Jarnagin WR, et al.,Cancer, (2003). Weber SM, et al., Journal of the American College of Surgeons,(2001).

8. In the Hopkins rapid autopsystudy, what percent of patients with pancreatic cancer died with "locallydestructive" disease?


(A) 10


(B) 20


(C) 30


(D) 40

Key: C Rationale: According to this rapid autopsy series, locallydestructive disease was found in 30% of patients. References:Iacobuzio-DonohueC., et al, J Clin Oncol.

46. Which of the following makespancreatic cancer unresectable?


(A) CA 19-9 of 250 ng/mL


(B) Invasion of the duodenum


(C) Encasement (270 degrees) of the superiormesenteric artery


(D) Involvement of peri-pancreatic lymphnodes in the field of resection

Key: C Rationale: Greater than 180 degreeSMA encasement is considered unresectable. Clearly resectable tumors must havean intact fat plane around the SMA. CA 19-9, invasion of the duodenum, andregional lymph node involvement in the field of resection do not make a tumorunresectable. References: NCCN Clinical PracticeGuidelines in Oncology.

60. According to the ACT IIstudy of definitive chemoradiation for anal cancer, the complete response rate(%) at 26 weeks was:


(A) 60.


(B) 70.


(C) 80.


(D) 90.

Key: D Rationale:The ACT II study isa 2 × 2 factorial trial which enrolled patients with histologically confirmedsquamous-cell carcinoma of the anus without metastatic disease from 59 centresin the UK. Patients were randomly assigned to one of four groups, to receiveeither mitomycin (12 mg/m(2) on day 1) or cisplatin (60 mg/m(2) on days 1 and29), with fluorouracil (1000 mg/m(2) per day on days 1-4 and 29-32) andradiotherapy (50.4 Gy in 28 daily fractions); with or without two courses ofmaintenance chemotherapy (fluorouracil and cisplatin at weeks 11 and 14). Witha median follow-up of 5.1 years, maintenance chemotherapy either regimens didnot improve progression-free survival compared with no maintenancechemotherapy.

63. The RTOG 9704 pancreaticcancer study demonstrated that:


(A) adjuvant chemoradiationimproves OS.


(B) neoadjuvant chemoradiationimproves OS.


(C) adjuvant gemcitabine and5-FU following chemoradiation show similar efficacy.


(D) gemcitabine and 5-FU showedsimilar efficacy as a radiosensitizer during chemoradiation.

Key: C Rationale: RTOG 9704 tested systemic 5FU vs gemcitabine in theadjuvant setting, after resection of pancreatic cancer. Patients received onemonth of chemotherapy, followed by chemoradiation followed by an additional 3months of chemotherapy. The chemoradiation for all patients was infusional 5FUwith a total dose of 5040 cGy of radiation. Median OS was 20.5 months withgemcitabine and 17.1 months with 5FU (P=0.12). References:Regine WF, et al,Ann Surg Oncol, (2011)

75. According to INT 0116,adjuvant chemoradiotherapy for gastric cancer is MOST appropriate for which ofthe following?


(A) pT3N1 disease, D0dissection, diffuse type


(B) pT3N1 disease, D1dissection, diffuse type


(C) pT1N0 disease, D1dissection, intestinal type (D) pT3N1 disease, D1dissection, intestinal type

Key: D Rationale: Most patients on the INT0116 trial had a D1 or lessdissection. Although adjuvant chemoradiation is an appropriate option forpatients with D2 dissection, subsequent trials in D2 patients have since shownthat adjuvant chemotherapy alone can also achieve comparable improvements insurvival. Therefore, adjuvant chemoradiation is especially appropriate forD1-resected patients, since the chemotherapy-alone regimens do not necessarilyapply. The updated analysis of INT0116 indicated that there was no benefit tochemoradiation in diffuse-type cancer. T1N0 patients were not eligible forinclusion on the INT0116 trial. References:Smalley, SR, J ClinOncol, (2012).

129. Which statement isMOST accurate for a 50-year-old woman with a gastric MALT?


(A) Multiagentchemotherapy and IFRT


(B) Local RT 30 Gy forH. Pylori negative


(C) Multiagentchemotherapy for H. Pylori negative


(D) H. Pylori treatmentand monitoring for at least 6 months before considering RT

Key: B Rationale: Anti-H. Pylori therapyis the most effective for H. Pylori positive cases. Radiotherapy is one of themost effective modality in treating gastric MALT for H. Pylori negative casesor for persistent disease.

142. What is the AJCCstage for a 5 cm adenocarcinoma starting from the GE junction, extending intothe gastric cardia, with extension into but not through the muscularis propriaand three suspicious peritumoral nodes?




(A) T2N1 gastric cancer


(B) T2N2 gastric cancer


(C) T2N2 esophagealcancer


(D) T3N2 esophagealcancer

Key: C Rationale: In the 7th edition AJCCstaging manual, cancers involving the GE junction and extending into thestomach are staged as esophageal cancers if the epicenter of the tumor iswithin 5cm of the GE junction. Extension into muscularis propria represents T2disease, and involvement of 3-6 nodes represents N2 disease. References: Rice, TW, Ann SurgOncol, (2010).

146. What were theresults of NSABP R-04, which evaluated the use of four chemotherapy regimenswith preoperative radiation therapy for Stage II and III rectal cancer?


(A) Toxicities weresimilar with the addition of oxaliplatin to capecitabine.


(B) The addition ofoxaliplatin to capecitabine improved sphincter sparing surgery.


(C) Capecitabine wassuperior to continuous infusion 5-FU for surgical downstaging.


(D) The addition ofoxaliplatin to capecitabine did not improve pathological complete responserates.

Key: D Rationale: Patients with clinicalstage II or III rectal cancer who were undergoing preoperative RT were randomlyassigned to: continuous intravenous infusional fluorouracil with or withoutintravenous oxaliplatin or oral capecitabine with or without oxaliplatin.Before random assignment, the surgeon indicated whether the patient waseligible for sphincter-sparing surgery based on clinical staging. 1,608patients were randomly assigned. No significant differences in the rates ofpCR, sphincter-sparing surgery, or surgical downstaging were identified betweenthe CVI FU and capecitabine regimens or between the two regimens with orwithout oxaliplatin. Patients treated with oxaliplatin experiencedsignificantly more grade 3 or 4 diarrhea.

147. What is theobserved rate of pathologic complete response after chemoradiation foresophageal cancer?


(A) 10-15%


(B) 25-30%


(C) 40-45%


(D) 55-60%

Key: B Rationale: The rate of pathologiccomplete response was 29% in the chemoradiation arm of the CROSS trial. The U.Michigan trial (Urba SG, J Clin Oncol 2001; 19:305-313) had a 28% pCR rate. AnEORTC trial (Bosset J, New Engl J Med 1997; 337:161-70) had a 26% pCR rate. References: Van Hagen, P. et.al, NewEngl J Med, (2012).

150. Which of thefollowing represents a dosimetric predictor of duodenal toxicity after IMRT fortreatment of para-aortic nodes in gynecologic cancers?


(A) V40 > 15 cm3


(B) V45 < 15 cm3


(C) V50 < 15 cm3


(D) V55 > 15 cm3

Key: D Rationale: 3-year actuarial ratesof duodenal toxicity with V55 above and below 15 cm3 were 48.6% and 7.4%,respectively (p <.01) In Cox, univariate analysis of dosimetric variables,V55 was associated with duodenal toxicity (P=0.29). In recursive partitioninganalysis, V55 less than 13.94% segregated all patients with duodenal toxicity. References: Verma, et. al,Dosimetric predictors of duodenal toxicity after intensity modulated radiationtherapy for treatment of the para-aortic nodes in gynecologic cancer, (2014).

166. Regarding NSABPR-03:


(A) there was noimprovement in DFS with preoperative chemoradiation.


(B) there was asignificant improvement in OS with preoperative chemoradiation.


(C) the chemotherapyused concurrently with radiation was 5-FU and leucovorin.


(D) it comparedpreoperative radiotherapy to preoperative chemoradiation therapy for rectalcancer.

Key: C Rationale: Patients with clinicalT3 or T4 or node-positive rectal cancer were randomly assigned to preoperativeor postoperative chemoradiotherapy. Chemotherapy consisted of fluorouracil andleucovorin with 45 Gy in 25 fractions with a 5.40-Gy boost within the originalmargins of treatment. The 5-year DFS for preoperative patients was 64.7% v53.4% for postoperative patients (P = .011). The 5-year OS for preoperativepatients was 74.5% v 65.6% for postoperative patients (P = .065). A completepathologic response was achieved in 15% of preoperative patients. Nopreoperative patient with a complete pathologic response has had a recurrence.

171. Among elderlywomen, which of the following cancers confers the HIGHEST risk of pelvicfracture following pelvic irradiation?


(A) Anal


(B) Colon


(C) Rectal


(D) Cervical

Key: A Rationale: Pelvic irradiationsignificantly increases the long-term risk of pelvic fractures. A SEER databasestudy (Baxter et al, JAMA (294(20):2587-93, 2005) found that women whounderwent radiation therapy were more likely to have a pelvic fracture thanwomen who did not undergo radiation therapy (cumulative 5-year fracture rate,14.0% vs 7.5% in women with anal cancer, 8.2% vs 5.9% in women with cervicalcancer, and 11.2% vs 8.7% in women with rectal cancer). The impact ofirradiation varied by cancer site: treatment for anal cancer was associatedwith a higher risk of pelvic fractures (hazard ratio, 3.16; 95% confidenceinterval, 1.48-6.73); than for cervical cancer (hazard ratio, 1.66; 95%confidence interval, 1.06-2.59); or rectal cancer (hazard ratio, 1.65; 95%confidence interval, 1.33-2.05). This is likely due to the inclusion ofinguinal lymph nodes in the radiation treatment field, which may inadvertentlyincrease dose to the femoral heads.

185. For squamous cellcarcinoma of the esophagus, what is the expected benefit of surgical resectionafter achieving a clinical response to concurrent chemoradiotherapy?


(A) Decreased localrecurrence, improved survival


(B) Decreased localrecurrence, unchanged survival


(C) Unchanged localrecurrence, improved survival


(D) Unchanged localrecurrence, unchanged survival

Key: B Rationale: In two randomizedtrials of chemoradiation with or without surgery (the other is Stahl M, J ClinOncol 2005; 23:2310-2317), the addition of surgery led to no significantimprovement in overall survival, but a decrease in rate of locoregionalrelapse. The Bedenne trial included mostly squamous carcinoma patients and theStahl trial only included squamous carcinoma, therefore these conclusions arenot necessarily applicable to adenocarcinoma. References: Bedenne, L. et. al, JClin Oncol, (2007) -

192. Forlocally advanced rectal cancer, a UK + NCIC trial comparing preoperative RT (25Gy/5fx) to selective postoperative chemoRT (45 Gy/25 fx) showed that:


(A) TMEreduced local recurrence.


(B)preoperative therapy improved OS.


(C)preoperative therapy improved DFS.


(D) negativecircumferential margins were improved with preoperative therapy.

Key: CRationale:This study found a reduction of 61% in the relative risk of local recurrencefor patients receiving preoperative radiotherapy (hazard ratio [HR] 0•39, 95%CI 0•27–0•58, p<0•0001), and an absolute difference at 3 years of 6•2% (95%CI 5•3–7•1) (4•4% preoperative radiotherapy vs 10•6% selective postoperativechemoradiotherapy). It showed a relative improvement in disease-free survival of24% for patients receiving preoperative radiotherapy (HR 0•76, 95% CI0•62–0•94, p=0•013), and an absolute difference at 3 years of 6•0% (95% CI5•3–6•8) (77•5% vs 71•5%). Overall survival did not differ between the groups(HR 0•91, 95% CI 0•73–1•13, p=0•40). The rate of circumferential resectionmargin involvement and the proportion of patients with pathological stage IIIdisease did not differ between the two groups (p=0•12 and p=0•29,respectively). 93% of patients had TME.

206.According to ACT II trial, what is the impact of maintenance chemotherapy on OSfollowing definitive chemoradiation for anal cancer?


(A) improvedwith 5-FU and Cisplatin


(B) noimpact with 5-FU and Cisplatin


(C) improvedwith 5-FU and Mitomycin-C


(D) noimpact with 5-FU and Mitomycin-C

Key: BRationale:The ACT II study is a 2 × 2 factorial trial which enrolled patients withhistologically confirmed squamous-cell carcinoma of the anus without metastaticdisease from 59 centres in the UK. Patients were randomly assigned to one offour groups, to receive either mitomycin (12 mg/m(2) on day 1) or cisplatin (60mg/m(2) on days 1 and 29), with fluorouracil (1000 mg/m(2) per day on days 1-4and 29-32) and radiotherapy (50.4 Gy in 28 daily fractions); with or withouttwo courses of maintenance chemotherapy (fluorouracil and cisplatin at weeks 11and 14). With a median follow-up of 5.1 years, maintenance chemotherapy eitherregimens did not improve progression-free survival compared with no maintenancechemotherapy.

230. In the originalGITSG 9173 study of adjuvant chemoradiation, chemoradiation with 5-FU improved:


(A) median OS from 11months to 20 months.


(B) median OS from 22months to 40 months.


(C) progression-freesurvival from 11 months to 20 months.


(D) progression-freesurvival from 22 months to 40 months.

Key: A Rationale: The original GITSG 9173trial of adjuvant chemoradiation showed a survival benefit in the 21 patientsrandomized to adjuvant chemoradiation over the 22 patients randomized to noadjuvant treatment. Adjuvant treatment was weekly bolus 5FU with radiationdelivered in two 2000 cGy two-week intervals with a two-week break in between.Median survival was 20 months with chemoradiation and 11 months withobservation. (P=0.03) References: Kalser MH, EllenbergSS., Arch Surg, (1985).

234. In thepostoperative management of rectal cancer, the NCCTG 86-47-51 trialdemonstrated that:


(A) semustine and 5-FUbefore and after chemoradiation improved OS.


(B) bolus 5-FU causedmore hand-foot syndrome and diarrhea compared to protracted venous infusion5-FU.


(C) bolus 5-FUconcurrent with RT reduced local tumor relapse rates compared to protractedvenous infusion 5-FU.


(D) protracted venousinfusion 5-FU with concurrent RT had superior OS compared to bolus 5-FU withRT.

Key: D Rationale: Patients who received aprotracted infusion of fluorouracil had a significantly increased time torelapse (P = 0.01) and improved survival (P = 0.005), and reduced tumor relapserates. There was no evidence of a beneficial effect in the patients whoreceived semustine plus fluorouracil. PVI resulted in more diarrhea and bolus5-FU resulted in more leukopenia.

261. According to RTOG98-11, the 5-year OS for patients with T4, node positive anal canal cancer is: (A) 20%. (B) 40%. (C) 60%. (D) 80%.

Key: B Rationale: The long-term update ofRTOG 98-11 found that concurrent chemoradiation with 5-FU and MMC had asignificant impact of DFS and OS over 5-FU and cisplatin. In a recent secondaryanalysis, TN category has a statistically significant impact on overallsurvival, disease-free survival, and colostomy failure. Among all patients withT4, node positive disease, the 5-year overall survival was 42%.

274. In the KoreanARTIST trial of adjuvant therapy in gastric cancer resected with D2 dissection,what was the impact of chemoradiation on DFS?


(A) Benefit in allpatients


(B) No benefit in anysubset


(C) Benefit innode-negative patients only


(D) Benefit innode-positive patients only

Key: D Rationale: There was nostatistically significant benefit in disease-free-survival with chemoradiationin the ARTIST trial as a whole. However, in the subset of patients withnode-positive disease (86% of the cohort), DFS was significantly improved. References: Lee, J., J Clin Oncol,(2012).

281. What is theappropriate clinical stage of a rectal adenocarcinoma that invades through themuscularis propria, with 4 suspicious lymph nodes on imaging?


(A) T3N1, stage IIIB


(B) T3N1, Stage IIIC


(C) T3N2, Stage IIIB


(D) T3N2, Stage IIIC

Key: C Rationale: T3= invasion throughmuscularis propria N1=1-3 LN N2=4 or more LN, N2a= 4-6 LN, N2b = 7 or more LNT3N2a= IIIB.

284. Regarding earlystage NSCLC eligible for treatment with SBRT:


(A) local controldecreases if more than 5 fractions are given.


(B) adjuvantchemotherapy improves DFS and OS following SBRT.


(C) there is level 1evidence that SBRT has equivalent OS in comparison to surgery.


(D) a BED of 100 Gy isan important dose to achieve high local control rates > 90%.

Key: D Rationale: According to Japanesedata, if the BED is ≥100 Gy, then the 5-yr LC rate is 92% and the 5-yr OS is71%. However, if the BED is <100 Gy, then the 5-yr LC rate is 57% and the5-yr OS is 30%. There is no data suggesting a benefit of adjuvant chemotherapyfollowing SBRT. There are no phase III trials that have been reported to datecomparing surgery and SBRT for early lung cancer.